U world fundamentals 7/12/21

Ace your homework & exams now with Quizwiz!

a) "Can you lock your dresser drawer?" Children are naturally curious and attracted to medicine, especially if it is sweet and syrupy like many over-the-counter cold products. They usually find medicines when exploring their environment and "getting into everything" when no one is watching. Children may find medicine in a parent's coat pocket or purse, under a counter cabinet, or on a nightstand. Even if a drug is stored in a place that seems out of reach, children can climb on a chair or stool to reach it. Medications are the leading cause of child poisoning. The best preventative measures include placing all medications out of sight, placing them in a drawer or cabinet with a childproof lock, and putting them away after each use (option 1). (option 2) Advising a parent/caregiver to ensure that medicine containers have childproof caps is an appropriate instruction; however, it is not the priority response in this situation. (option 3) Storing medicines in a dresser drawer is not a safe plan unless the drawer can be locked. (option 4) Although it is impossible for a parent or caregiver to watch a child every minute of the day, toddlers need adult supervision when active and exploring their environment. Educational objective: The most important strategy to prevent accidental drug overdoses in children is teaching parents and caregivers to keep medicines out of sight, in a locked drawer or cabinet. Parents/caregivers should also be advised to put drugs away after each use.

A 2-year-old who swallowed an overdose of adult cough syrup is being discharged from the emergency department. The parent says to th nurse, "From now on, I'm going to store all medicines in my top dresser drawer." Which is the best response by the nurse? a) "Can you lock your dresser drawer?" b) "Make sure all of your medicines have childproof caps." c) "That sounds like a safe plan." d) "You need to keep an eye on your child at all times."

a) Check the urethral catheter and drainage tubing Obstruction (eg, clots, sediment), kinking/compression of catheter tubing, bladder spasms, and improper catheter size can cause leakage of urine from the insertion site of an indwelling urinary catheter. The nurse's first action should be to assess for a mechanical obstruction by inspecting the catheter tubing (option 1). These interventions may alleviate obstruction: - Remove kinking or compression of the catheter or tubing. - Attempt to dislodge a visible obstruction by milking the tubing. This involves squeezing and releasing the full length of the tubing, starting from a point close to the client and ending at the drainage bag. If these interventions fail, the nurse should then notify the health care provider (HCP) (option 3). (option 2) Irrigation is usually avoided as pus or sediment can be washed back into the bladder; however, it is sometimes prescribed to relieve an obstruction to urine flow. If there is a discrepancy in expected urine output compared with fluid intake, a blockage is suspected and a bladder scan is then performed to confirm the presence of urine in the bladder. (option 4) The client has the recommended size of catheter and balloon for an adult male. The HCP may prescribe removal and reinsertion of a different-size catheter if other measures fail to relieve obstruction. Educational objective: If leakage of urine is observed from the insertion site of an indwelling urinary catheter, the nurse should assess for obstruction, kinking, or compression of the catheter or drainage tubing; bladder spasms; and improper catheter size.

A 55-year-old male client has a 16-Fr indwelling urethral catheter with a 5 mL balloon inserted to relive postoperative urinary retention. The nurse observes urine leaking from the insertion site, past the catheter. What is the nurse's first action? a) Check the urethral catheter and drainage tubing b) Irrigate the catheter with 30 mL of sterile normal saline c) Notify the health care provider d) Remove and reinsert the next-larger-size catheter

d) Prepares to administer IVPB potassium chloride via gravity infusion for a client with hypokalemia Treatment of hypokalemia may require an IV infusion of potassium chloride (KCL). The infusion rate should not exceed 10 mEq/hr (10 mmol/hr). Therefore, IVPB KCL must be given via an infusion pump so the rate can be regulated. IV KCL should be diluted and never given in a concentrated amount. Furthermore, too rapid infusion can cause cardiac arrest. The charge nurse would need to intervene if the new nurse was attempting to administer IVPB KCL via gravity infusion instead of a pump. (option 1) Hydromorphone is indicated for moderate to severe pain. A pain rating of 7 would warrant its administration. (option 2) Occasional premature ventricular contractions (PVCs) in the normal heart are not significant. PVCs in the client with coronary artery disease or myocardial infarction indicate ventricular irritability and may lead to life-threatening dysrhythmia such as ventricular tachycardia. (option 3) With the complete removal of the lung in a pneumonectomy, the client should be positioned on the surgical side to promote adequate expansion and ventilation of the remaining lung. Educational objective: IV infusion of potassium must be administered via a pump to prevent too rapid infusion, which could cause cardiac arrest.

A charge nurse is monitoring a newly licensed registered nurse. What action by the new nurse would warrant intervention by the charge nurse? a) Administers hydromorphone 1 mg to a client who rates pain at 7 on a 1 to 10 scale b) Notifies physician of occasional premature ventricular beats in a client with myocardial infarction c) Positions a postoperative pneumonectomy client on the affected side d) Prepares to administer IVPB potassium chloride via gravity infusion for a client with hypokalemia

c) Inject through the clothing into thigh and hold in place for 10 seconds The EpiPen is designed to be administered through clothing with a swing and firm push against the mid-outer thigh until the injector clicks. The position should be held for 10 seconds to allow the entire contents to be injected (options 3). The site should be massaged for an additional 10 seconds. Timing is essential in the delivery of epinephrine during an anaphylactic reaction. The nurse should administer the medication immediately on the playground without removing the child's clothing. Any delays can cause client deterioration and make maintenance of a patent airway difficult (option 4). (option 1) The EpiPen should be injected into the mid-outer thigh, not the upper arm. (option 2) IV epinephrine is not administered outside the hospital setting. It requires cardiac monitoring and is indicated in clients with profound hypotension (shock) or those who do not respond to intramuscular epinephrine and fluid resuscitation. Educational objective: The EpiPen is designed to be delivered through clothing in the mid-outer thigh area. The nurse should not delay anaphylaxis treatment by attempting to remove the client's clothing.

A child on the playground is experiencing an anaphylactic reaction. The school nurse arrives with an EpiPen. The weather is cold and the child is wearing several layers of clothing. How should the nurse proceed with the EpiPen? a) Inject into the upper arm where the sleeve can be pulled up b) Inject into the most accessible vein c) Inject through the clothing into thigh and hold in place for 10 seconds d) Take the child inside, remove excess clothing, and inject into the thigh

2 mL Furosemide is a diuretic medication used to treat fluid overload in children with congenital heart disease. Educational objective: To calculate the volume per dose of furosemide, the nurse should first identify the prescribed dose (eg, 1 mg/kg/dose) and available medication (eg, 10 mg/mL) and then convert to volume in milliliters per dose (eg, 2 mL).

A child with congenital heart disease weighing 44 lb is prescribed furosemide 1 mg/kg PO every 8 hours. It is available as an oral solution of 10 mg/mL. How many milliliters (mL) of furosemide should the nurse administer to the client each dose? Record your answer using a whole number. Answer: ________________

b) Exchange the IV pump with a different one IV infusion pumps display an occlusion alarm when IV solution cannot be infused due to pressure in the line. Common causes of occlusion include clamped or kinked IV tubing, clotting in the IV catheter, and kinking in the IV catheter with extremity movement (eg, elbow, wrist). The nurse should assess the tubing and IV site and flush the IV catheter to check patency. In the absence of identifiable occlusion, an alarming IV pump should be exchanged for a different one (option 2). Malfunctioning equipment may harm the client and should be removed from the care area. The malfunctioning equipment is labeled as out of service and is sent for maintenance. (option 1) An IV pump alarm does not indicate that a medication is no loner needed. The nurse should replace a malfunctioning pump and restart the medication. (option 3) An IV catheter that has no symptoms of occlusion (ie, resistance to flushing) or infiltration (eg, swelling, coolness, pain) does not need to be replaced (option 4) IV pump infusion is more accurate than gravity drip. IV pumps are required when administering high-risk IV medications (eg, heparin, insulin, potassium). IV potassium should never be administered by gravity as it may cause lethal arrhythmia if administered too quickly. Educational objective: If an IV infusion pump displays an alarm without an identifiable problem, the nurse should replace the pump. Malfunctioning equipment may harm the client and should be removed from the care area. The equipment is labeled as out of service and sent for maintenance.

A client is receiving IV potassium. The IV pump displays an occlusion alarm. The tubing is free of occlusions, and the IV flushes easily without symptoms of infiltration. Which action should the nurse take next? a) Discard potassium and document administration of a partial dose b) Exchange the IV pump with a different one c) Insert a new IV catheter in a different location d) Remove the pump and administer medication by gravity drip

d) Stop the blood transfusion e) Using new tubing, infuse normal saline into the vein a) Administer prescribed vasopressor b) Collect urine specimen c) Document the occurrence It is important for the nurse to remain with the client for 15 minutes after starting a blood transfusion to monitor for signs of a reaction. These signs include fever, chills, nausea, vomiting, pruritus, hypotension, decreased urine output, back pain, and dyspnea. The client may report a variety of symptoms ranging from none to a feeling of impending doom. If signs of a transfusion reaction occur, the nurse should: - Stop the transfusion immediately (option 4) - Using new tubing, infuse normal saline to keep the vein open (option 5) - Continue to monitor hemodynamic status and notify the health care provider and blood bank. -Administer any emergency or prescribed medications to treat the reaction; these may include vasopressors, antihistamines, steroids, or IV fluids (option 1) - Collect a urine specimen to be assessed for a hemolytic reaction (option 2) - Document the occurrence and send the remaining blood and tubing set back to the blood bank for analysis (option 3) Educational objective: If signs or symptoms of a blood transfusion reaction occur, the nurse should stop the infusion immediately and use new tubing to keep the vein open with normal saline. The nurse should continue to monitor the client's hemodynamic status, and administer prescribed drugs. The nurse should also collect a urine specimen to be assessed for a hemolytic reaction

A client is receiving a blood transfusion. Fifteen minutes after the transfusion starts, the nurse notes a drop in blood pressure from 110/70 to 84/50 mmHg. The client reports "feeling a little cold." Based on this assessment, in what order should the nurse complete the following actions? All options must be used. Unordered Options: a) Administer prescribed vasopressor b) Collect urine specimen c) Document the occurrence d) Stop the blood transfusion e) Using new tubing, infuse normal saline into the vein

c) Explore the client's abilities and motivation to perform care at home Self care is a critical component of health. However, barriers to self care are multifactorial, and include: - knowledge (lack of experience, cognitive abilities) - skills/supplies (lack of dexterity, experience, financial barriers) - motivation (lack of assumed threat to health, denial, hopelessness) The nurse must assess for adequate knowledge and ability to perform self-care activities and the desire to complete such activities (option 3). Once the barriers have been identified, the nurse can work with the client to create an individualized plan to meet health care needs. (option 1) Without understanding the barriers to self care, the nurse cannot identify proper resources to assist the client in meeting needs. Placement for skilled nursing may be excessive for a client who lives independently. (option 2) Education on tissue death may be perceived as threatening and not therapeutic. (option 4) Financial resources or supplies may not be the barrier; therefore, this intervention may not effectively assist the client in performing self care successfully. Educational objective: The nurse must assess a client's knowledge, skills, and motivation to identify barriers to self care. Through this identification, the nurse can help develop an individualized plan to meet health care needs.

A client is seen in the clinic for the third time for a non healing, infected diabetic foot ulcer. The client is able to verbalize the correct procedure for wound care but reports not adhering to the ordered routine at home. What intervention does the nurse prioritize to promote proper self care? a) Assess the client's feelings about placement at a skilled nursing facility for care b) Educate the client on the risks of tissue death is not properly cared for at home c) Explore the client's abilities and motivation to perform care at home d) Provide the client with supplies needed to change dressings as recommended

d) Verify the client's activity prescription A client newly admitted from a surgical procedure may have activity restrictions or bed rest prescribed for a certain period. Before assisting the client to the chair, the nurse needs to verify the activity level prescribed by the health care provider. Getting the client out of bed too early could cause injury to the surgical site or result in a fall. (option 1) A client who had knee surgery will likely be unable to bear any weight on the affected limb. Depending on the client's size, it may be prudent for the nurse to get additional help. This could be requested after the activity prescription has been verified. (option 2) Assistance with ambulation is often delegated to unlicensed assistive personnel (UAP); however, the nurse should verify the prescription first. It would also be prudent to have the UAP assist the nurse as this is the client's first time up after surgery. (option 3) The nurse should assess the client's pain level before providing pain medication Educational objective: The nurse should verify activity prescriptions before getting clients out of bed after surgery or a procedure. The nurse should be present when these clients begin ambulating and may need assistance from another nurse or unlicensed assistive personnel.

A client newly returned to the unit after knee surgery asks the nurse for assistance to a chair. What action should the nurse implement first? a) Ask another nurse for help b) Delegate the task to unlicensed assistive personnel c) Premedicate the client for pain d) Verify the client's activity prescription

900 mL A transurethral prostatectomy (TURP) is a surgical prostate-removal procedure commonly performed for male clients with prostate cancer. Following a TURP, clients typically receive continuous bladder irrigation (CBI) with a sterile, isotonic solution (eg, normal saline) via indwelling urinary catheter. CBI prevents bladder obstruction by large blood clots in the bladder or urethra. Monitoring urine output in clients receiving CBI can be challenging because there is continuous output from the irrigation. To calculate net urine output, the nurse should obstruct subtract the irrigation input from the total catheter output. Educational objective: Continuous bladder irrigation is a therapy commonly used to prevent bladder obstruction by blood clots after a prostatectomy. To calculate the net urine output in a client with continuous bladder irrigation, the nurse should subtract the total amount of irrigating solution infused from the total amount of catheter output.

A client postoperative from a transurethral prostatectomy has a triple-lumen, indwelling urinary catheter and is receiving continuous bladder irrigation of sterile normal saline solution at 175 mL/hr. The nurse empties the urine drainage bag for a total of 2300 mL at the end of the 8-hour shift. How many milliliters (mL) should the nurse document as the net urine output for the shift? Record your answer using a whole number. Answer: ____________

a) "Always ask the health care provider or pharmacist before taking OTC medications." b) "Ingredients in some OTC medications may interact with prescription medications." d) "Remember to discuss all medications, herbs, and supplements you take with your health care providers." e) "Taking OTC medications can sometimes hide symptoms of a serious disease or illness." Over-the-counter (OTC) medications are available without a prescription and are used to treat common illnesses. It is estimated that nearly four times as many health conditions are independently managed with OTC medications as are managed under the supervision of a health care provider (HCP). Prior to taking OTC medications, the client should talk with an HCP or pharmacist, particularly if already taking prescribed medications (option 1). Even when taken as directed by the OTC medication label, interactions and adverse effects may occur when used in combination with prescription medications (option 2). All medications, herbal products, and supplements must be discussed with HCPs so that they can be reconciled and considered before changing or adding new treatments (option 4). When OTC medications are used to manage symptoms (eg, ibuprofen for back pain), the diagnosis and treatment of serious underlying medical conditions (eg, malignancy) may be delayed (option 5). (option 3) Herbal products and supplements, although they are prepared from plants or "all-natural" substances, may contain compounds that interact with prescription medications. These interactions may cause increased or decreased prescription medication effect, serious adverse effects, and medication toxicities. Educational objective: Nurses should instruct clients to talk with a health care provider (HCP) or pharmacist before taking over-the-counter medications as they may interact with prescription medications or hide symptoms of a serious condition. All medications, herbal products, and supplements taken should be discussed with HCPs.

A client who has been prescribed several medications asks, "Can I take over-the-counter (OTC) medications with my prescriptions?" Which of the following statements by the nurse is appropriate? Select all that apply. a) "Always ask the health care provider or pharmacist before taking OTC medications." b) "Ingredients in some OTC medications may interact with prescription medications." c) "It is best to avoid OTC medications, but herbal and supplement products are usually safe." d) "Remember to discuss all medications, herbs, and supplements you take with your health care providers." e) "Taking OTC medications can sometimes hide symptoms of a serious disease or illness."

a) Assess abdominal dissension every 4 hours c) Keep head of the bed > 30 degrees d) Maintain endotracheal cuff pressure e) Use caution when administering sedatives Clients who are critically ill are at increased risk for aspiration of oropharyngeal secretions and gastric contents, particularly when they are receiving enteral feedings. Nursing interventions to reduce aspiration risk for clients receiving enteral tube feedings include: - Assess client for gastrointestinal intolerance to feedings every 4 hours by monitoring gastric residual and assessing for abdominal distension, abdominal pain, bowel movements, and flatus (option 1) - Assess feeding tube placement at regular intervals - Keep head of the bed > 30 degrees, with 30-45 degrees being optimal to reduce gastroesophageal reflux and aspiration risk unless otherwise indicated (option 3) - Keep endotracheal cuff inflated at appropriate pressure (about 25 cm H2O) for incubated clients, as low cuff pressure increases the risk for aspirating oropharyngeal secretions and/or gastric contents (option 4) - Suction any secretions that may have collected above the endotracheal tube before deflating the cuff if deflation is necessary - Use caution when giving sedatives and frequently monitor for over-sedation, which can slow gastric emptying and reduce gag reflex (option 5) - Avoid bolus tube feedings for clients at high risk for aspiration (option 2) Gastric residual should be checked no less than every 4 hours in intubated clients. Educational objective: Precautions to prevent aspiration in the client receiving continuous tube feedings include assessing for gastric intolerance (ie, residual, distension) every 4 hours, keeping the head of the bed at >30 degrees, using sedation cautiously, and regular assessment of tube placement. If the client is intubated, the nurse should also keep the endotracheal tube cuff inflated and suction appropriately.

A client who is incubated and on mechanical ventilation is receiving continuous enteral tube feedings at 30 mL/hr via a small-bore nasogastric tube. Which actions should the nurse take to prevent aspiration in this client? Select all that apply. a) Assess abdominal dissension every 4 hours b) Check gastric residual every 12 hours c) Keep head of the bed > 30 degrees d) Maintain endotracheal cuff pressure e) Use caution when administering sedatives

b) The hand is held slightly below elbow level A sling is used to support the shoulder after a fracture, dislocation, injury, or surgery. Commercially made slings are used almost exclusively. They have a sleeve that fits around the injured extremity and extends above the elbow and adjustable straps to provide a snug and comfortable fit around the waist and neck. To prevent injury and provide proper support of the affected extremity, the nurse should evaluate the proper fit of the sling by assessing for the following factors: - Elbow is flexed at 90 degrees to support the forearm, prevent swelling, and relieve shoulder pressure (option 1) - Hand is held slightly above the level of the elbow, through adjustment of the neck strap, to prevent venous pooling and edema (option 2) - Bottom of the sling ends in the middle of the palm with the fingers visible, to be able to assess circulation, sensation, and movement (option 3) - Sling supports the wrist joint with the thumb facing upward or inward toward the body, to maintain proper alignment (option 4) - Skin irritation, which can occur under the sling and around the neck if the strap is too tight. Educational objective: To ensure proper shoulder sling fit, the nurse should assess for the following: - Elbow is flexed at 90 degrees - Hand is held slightly above the level of the elbow - Bottom of the sling ends in the middle of the palm with the fingers visible - Sling supports the wrist joint

A client with a dislocated shoulder is prescribed a shoulder sling. The nurse applies the sling and evaluates the fit before discharge from the emergency room. Which assessment finding indicates an incorrect fit? a) The elbow is flexed at 90 degrees b) The hand is held slightly below elbow level c) The sling ends in the middle of the palm with fingers visible d) The sling supports the wrist

c) Partially or fully deflate the cuff A tracheostomy tube with inflated cuff is used in clients who are at risk for aspiration (eg, who are unconscious or on mechanical ventilation). However, an inflated cuff is uncomfortable for clients who are awake because it is difficult to swallow or talk. The cuff is deflated when the client is improving, is determined not to be at risk of aspiration, and is awake. Before the cuff is deflated, the client is asked to cough (if possible) to expectorate the oropharyngeal secretions that have built up above the inflated cuff. In addition, suction is applied through the tracheostomy tube and then the mouth; the cuff is then deflated. Additional interventions to decrease the risk of aspiration include the following: -Having the client sit upright with the chin flexed slightly toward the chest - Monitoring for a wet or garbled-sounding voice - Monitoring for signs of fever (option 1) Inflating the cuff makes it difficult for a client who is awake to swallow and talk. In addition, more secretions can accumulate above the inflated cuff due to difficulty swallowing. The inflated cuff may not provide a 100% seal and the accumulated secretions can slide through it, causing aspiration. For these reasons, the deflated cuff is beneficial in awake clients with no risk of aspiration. (option 2) Having the client sit upright will help reduce the risk of aspiration. However, the chin should be flexed toward the chest; hyperextension of the neck increases the risk of aspiration. (option 4) There is no reason to give puréed foods just because the client has a tracheostomy. The client's diet should be determined by a swallow evaluation. Educational objective: The risk of aspiration in a conscious, alert, and oriented client with a tracheostomy can be reduced by partially or fully deflating the tracheostomy cuff, having the client in an upright position, monitoring for a wet cough or voice quality, and monitoring vital signs.

A client with a tracheostomy is alert and oriented and able to tolerate oral intake. Which action would be appropriate to reduce the client's risk of aspiration pneumonia? a) Fully inflate the cuff before feeding b) Have the client sit in an upright position with the neck hyperextended c) Partially or fully deflate the cuff d) Provide a modified diet of puréed foods

Answer: 125 mL/hr Educational objective: To calculate the infusion rate of potassium chloride, the nurse should first identify the prescribed dose (eg, 10 mEq/hr) and available medication (eg, 20 mEq/250 mL) and then convert to volume in milliliters per hour (eg, 125 mL/hr).

A client with hypokalemia is prescribed IV potassium chloride (KCl) to infuse at 10 mEq/hr. The pharmacy sends 20 mEq of KCl in 250 mL of D5W. To deliver the prescribed dose, the nurse sets the infusion pump at how many milliliter steps per hour (mL/hr)? Record your answer using a whole number. Answer:__________________

c) Step slightly behind client with feet apart, extend one leg, and let client slide against it to the floor To prevent injury to the nurse and the client if the client is falling, the nurse uses good body mechanics to try to break the fall and guide the client to the floor if necessary. These actions include: - Step slightly behind the client and place the arms under the axillae or around the client's waist - Place feet wide apart with knees bent- creates a broad base of support, provides stability, and reduces the risk for back injury to the nurse - Place one foot behind the other and extend the front leg- allows the nurse to bring the client backward by using the leg muscles to rock backward while supporting the client's weight - Let the client slide down the extended leg to the floor- lowers the client gently to the floor while keeping the client's head protected from injury (options 1 and 4) These actions do not provide close proximity to the client, a broad base of support, or a lower center of gravity to increase the nurse's stability and help prevent back injury. (option 2) These actions are appropriate for helping a client rise from the bed or chair but not for assisting a falling client to the floor. Educational objective: These nursing actions can help prevent injury if a client is falling while the nurse is assisting with ambulation: step slightly behind the client with feet apart and knees bent, place arms under the axillae or around the client's waist, place one leg behind the other and extend the front leg, and let the client slide down the extended leg to the floor.

A client with right-sided weakness becomes dizzy, loses balance, and begins to fall while the nurse is assisting with ambulation. Which nursing actions would best prevent injury to the client and nurse while guiding the client to a horizontal position on the floor? a) Step behind client with arms around waist, squat using the quadriceps, and lower client to the floor b) Step in front of client, brace knees and feet against the client's, and assist to the floor gently c) Step slightly behind client with feet apart, extend one leg, and let client slide against it to the floor d) Step 12 inches behind the client, support under axillae, tighten back, and lower client to the floor

a) Aneurysm clip b) Cardiac pacemaker d) Retained metal foreign body in eye Clients must be screened for contraindications before exposure to a magnetic field (MRI) as it can damage implanted devices or metallic implants. Absolute contraindications can preclude testing, and relative contraindications can pose a hazard to the client's devices or implants, affect the quality of the images, or cause discomfort. Absolute contraindications: - cardiac pacemaker (option 2) - implantable cardioverter defibrillator - cochlear implant - retained metallic foreign body, especially in organs such as the eye (option 4) Relative contraindications: -Prosthetic heart valve - Metal plate, pin, brain aneurysm clip, or joint prosthesis (option 1)- Some of these devices have nonferrous MRI-safe materials and should be verified. - Implantable device (eg, insulin pump, medication port) Other factors that can affect the client's eligibility include inability to remain supine for 30-60 minutes and claustrophobia; however, these concerns are often controllable (eg, sedation can be prescribed, open MRI machine can be used). (option 3) A colostomy is not contraindication for MRI. (option 5) Transdermal metal-containing medication patches (clonidine, nicotine, scopolamine, testosterone, or fentanyl) are not a contraindication for MRI. However, the nurse should remove the patch beforehand due to the risk of burns and replace after testing. Educational objective: Usual contraindications for MRI include implanted devices (eg, pacemaker, implanted cardioverter defibrillator, medication ports), certain metal implants (eg, plates, pins, brain aneurysm clips, joint prostheses), and presence of a retained metal foreign body. However, some of these devices are now manufactured with MRI-safe materials that should be verified.

A client with suspected foot osteomyelitis is scheduled for an MRI. Which findings should the nurse notify the health care provider about before the test? Select all that apply. a) Aneurysm clip b) Cardiac pacemaker c) Colostomy d) Retained metal foreign body in eye e) Transdermal testosterone patch

b) Ensure each client has a separate container to empty collection bag c) Keep catheter bag below the level of the bladder e) Use sterile technique when collecting a urine specimen Health care catheter-associated UTIs are prevalent among hospitalized clients with indwelling urinary catheters. Steps to prevent infections in clients with urinary catheters include the following: - Wash hands thoroughly and regularly - Perform routine perineal hygiene with soap and water each shift and after bowel movements - Keep drainage system off the floor or contaminated surfaces - Keep the catheter bag below the level of the bladder - Ensure each client has a separate, clean container to empty collection bag and measure urine - Use sterile technique when collecting a urine specimen - Facilitate drainage of urine from tube to bag to prevent pooling of urine in the tube or backflow into the bladder - Avoid prolonged kinking, clamping, or obstruction of the catheter tubing - Encourage oral fluid intake in clients who are awake and if not contraindicated - Secure the catheter in accordance with hospital policy (tape or Velcro device) - Inspect the catheter and tubing for integrity, secure connections, and possible kinks (option 1) Perineal hygiene is performed using soap and water only every shift and as needed. Routine use of antiseptic cleansers is not shown to prevent infection and may lead to the development of drug-resistant bacteria. (option 4) Routine irrigation with anti microbial solution or systemic administration of antimicrobials is not recommended for routine catheter care and infection prevention. Educational objective: Routine catheter care to prevent health care catheter-associated UTIs includes routine hand hygiene, cleansing the perineal area with soap and water routinely, keeping the catheter bag below the bladder and off the ground, keeping the catheter and tubing free of kinks and facilitating urine into the bag, and using sterile technique when collecting urine specimens.

A comatose client in the intensive care unit has an indwelling urinary catheter. Which action(s) should the nurse implement to reduce the incidence of catheter-associated urinary tract infections (UTIs)? Select all that apply a) Cleanse periurethral area with antiseptic every shift b) Ensure each client has a separate container to empty collection bag c) Keep catheter bag below the level of the bladder d) Routinely irrigate the catheter with anti microbial solution e) Use sterile technique when collecting a urine specimen

4.6 units/hr Educational objective: To calculate the hourly dose of regular insulin, the nurse should first identify the prescribed dose (eg, 0.2 units/kg/hr) and then convert to units per hour (eg, 4.6 units/hr).

A continuous regular insulin IV infusion 0.2 units/kg/hr is prescribed for a 10-year-old client who weighs 51 lb and has diabetes mellitus. How many units per hour (units/hr) would the nurse administer to this client? Record your answer using one decimal place. Answer: ___________

b) Maintaining the affected leg flat on the bed Cellulitis is inflammation of the subcutaneous tissue that is typically caused by bacterial infection (eg, Staphylococcus aureus, group A Streptococcus) resulting from an insect bite, cut, abrasion, or open wound. Cellulitis is characterized by redness, edema, pain and fever. Nurses caring for clients with cellulitis should ensure that the affected extremity is elevated when the client is sitting or lying down to promote lymphatic drainage. Flat or dependent positioning may worsen edema, which delays recovery and contributes to pain (option 2). In addition, clients with weeping or draining wounds must be protected from prolonged exposure to moist or soiled linens as this exposure promotes tissue injury and infection. (option 1) Applying warm compresses promotes circulation to the area of infection, alleviates discomfort, and helps reduce edema. (option 3) Daily marking and dating of reddened areas assist with monitoring improvement or worsening of the infection. Redness that progresses past the marked areas indicates ineffective antibiotic therapy and should be reported to the health care provider. (option 4) Although standard precautions are typically sufficient for cellulitis, a gown and gloves are worn when contact with body fluids (eg, urine, stool) or potentially infectious drainage is expected, such as during bathing. Educational objective: Nurses caring for clients with cellulitis should ensure that the affected extremity is elevated to reduce edema. Additional nursing interventions include applying warm compresses, monitoring the size of the cellulitis, and using personal protective equipment to prevent infection transmission.

A graduate nurse (GN) is caring for a client with right lower leg cellulitis that is seeping clear fluid. Which action by the GN requires intervention by the supervising nurse? a) Applying a warm compress to the affected extremity b) Maintaining the affected leg flat on the bed c) Marking and dating the reddened areas d) Wearing a gown and gloves while bathing the client

a) Open a sterile container of 4x4's using the outermost corner to peel back the cover. b) Pull glove off over the soiled dressing to encase it before disposal d) Wash hands prior to putting on gloves and after removing them The nurse is responsible for observing the home health aide periodically during delegated tasks. The aide should wash the hands prior to gloving and after glove removal (option 4). Sterile dressing supplies should be opened prior to the dressing change; this should be done by carefully peeling from the outermost corner of the package to expose the contents without contaminating the sterile product (option 1). A contaminated used dressing should be placed in impervious plastic or a paper bag before disposal in the household trash (option 2). (option 3) Unused sterile supplies should not be saves as it is not possible to ensure their sterility (option 5) Paper towels are not impervious and infectious waste from the dressing can seep through and into other items in the trash can. Educational objective: In the home care setting, infection control procedure for changing a dressing include washing the hands before and after gloving, opening sterile supplies carefully to avoid contamination, and placing old dressings inside a glove or plastic bag before disposal in the household trash.

A home health nurse is supervising a home health aide who is changing the dressing for a client with a chronic heel wound. Which actions by the aide indicate adherence to appropriate infection control procedures? Select all that apply. a) Open a sterile container of 4x4's using the outermost corner to peel back the cover. b) Pull glove off over the soiled dressing to encase it before disposal c) Save unused sterile 4x4's by taping original package shut for the next dressing change d) Wash hands prior to putting on gloves and after removing them e) Wrap soiled dressing in paper towels before disposing of it in the trash can

a) Aim the nozzle at the base of the fire b) Pull out the pin on the handle d) Squeeze the handle to spray e) Sweep the spray from side to side A small fire can quickly become very dangerous. During an emergency situation, such as a fire, anxiety can narrow a person's focus, causing hesitation or difficulty in responding to the situation, especially when operation of unfamiliar equipment (eg, fire extinguisher) is involved. The mnemonic PASS is often used to help people remember the steps used in operating a fire extinguisher: P- Pull the pin on the handle to release the extinguisher's locking mechanism A- Aim the spray at the base of the fire S- Squeeze the handle to release the contents/extinguishing agent S- Sweep the spray from side to side until the fire is extinguished (option 3) The extinguisher does not need to be shaken before use, and doing so would delay extinguishing the fire. Educational objective: PASS is a mnemonic to help people remember the steps used in operating a fire extinguisher: P- pull the pin; A- Aim the spray at the base of the fire; S- Squeeze the handle; and S- Sweep the spray.

A nurse is making a home visit when a fire starts in the client's kitchen trash can. The client has a fire extinguisher. The nurse should take which actions to properly operate the fire extinguisher? Select all that apply. a) Aim the nozzle at the base of the fire b) Pull out the pin on the handle c) Shake the canister prior to use d) Squeeze the handle to spray e) Sweep the spray from side to side

c) Potassium of 7.0 MEq/L (7.0 mmol/L) With the exception of client in end-stage renal disease, a serum potassium value >6.5 mEq/L (6.5 mmol/L) is any client who is walking and talking should raise the suspicion of an erroneously elevated serum potassium (pseudohyperkalemia) from poor hematology technique, hemolysis, or clotting. A serum potassium level of 7.0 mEq/L (7.0 mmol/L) would normally constitute a life-threatening electrolyte imbalance that would cause severe weakness or paralysis, unstable arrhythmias, and eventual cardiac arrest. An assessment would focus on the evaluating cardiac symptoms and muscle strength and be reported tot the health care provider (HCP). In this case, it is likely that a repeat blood draw would be prescribed. Pseudohyperkalemia can be avoided on the repeat blood draw by using heparin-impregnated hematology vials to prevent clotting, minimal use if a tourniquet and fist clenching, and use of a larger gauge needle for the sample. (option 1) This blood urea nitrogen (BUN) value is elevated (normal 6-20 mg/dL [2.1-7.1 mmol/L]) and could be related to kidney damage or dehydration. Therefore, it is not the most likely erroneous result. (option 2) Similar to the BUN level, this creatinine value is significantly elevated (normal 0.6-1.3 mg/dL [53-115 umol/L]). Further nursing assessment is indicated, with documentation and involvement of the HCP in evaluating the impact of this kidney damage on the client's health. (option 4) This sodium value is high (normal 135-145 mEq/L [135-145 mmol/L]) and requires further exploration. The nursing assessment should be documented and reported to the HCP. Educational objective: High serum potassium levels could be due to hemolysis or clotting during the blood draw. If a clinical assessment does not correlate with the laboratory values, repeat testing is needed.

A nurse is reviewing the most recent laboratory results of a client on the telemetry floor. The client is currently a symptomatic, and the telemetry monitor indicates sinus rhythm. Which of the following critical values is most likely due to laboratory error? a) Blood urea nitrogen (BUN) of 60 mg/dL (21.4 mmol/L) b) Creatinine of 4.0 mg/dL (354 umol/L) c) Potassium of 7.0 MEq/L (7.0 mmol/L) d) Sodium of 155 mEq/L (155 mmol/L)

b) Direct liquid medication toward the inside of the infant's cheek c) Hold the infant in a semi-reclining position during administration d) Measure and administer the medication using an oral syringe Giving oral medications to infants requires specialized techniques for safe administration. A plastic, disposable oral syringe can be used for accurate dosing and ease of delivery (option 4). Oral medication should be administered with the infant in a semi-reclining position, which is similar to the feeding position (option 3). This position promotes comfort, prevents aspiration, and may be better controlled by the nurse if the infant resists the medication. Liquid medications administered by oral syringe should directed toward the back and inside of the infant's cheek (option 2). The medication should be dispensed slowly in small amounts, allowing the infant to swallow between squirts to prevent aspiration. (option 1) Medications are never mixed in a bottle of infant formula as this can affect the taste and the infant may then refuse the formula in the future. In addition, if the infant does not complete the full feed, underdosing will occur. (option 5) Pinching the nose shut during medication administration may cause aspiration. The infant's mouth should be opened by applying gentle pressure to the chin or cheeks. Educational objective: Disposable oral syringes are the preferred tool to administer oral medications to infants. Infants should be held in a semi-reclining position, and medications should be given slowly in small amounts directed toward the back and inside of the cheek.

A nurse is teaching a parent of an infant about administration of an oral medication. What should be included in the teaching? Select all that apply. a) Add the medication to the bottle of formula before feeding b) Direct liquid medication toward the inside of the infant's cheek c) Hold the infant in a semi-reclining position during administration d) Measure and administer the medication using an oral syringe e) Open the infant's mouth by gently pinching the nose shut

7.5 mL Educational objective: To calculate the volume per dose of ibuprofen, the nurse should first identify the prescribed dose (eg, 5 mg/kg/dose) and available medication (eg, 20 mg/mL) and then convert to volume in milliliters per dose (eg, 7.5 mL).

A pediatric client weighing 66 lb is prescribed ibuprofen 5 mg/kg by mouth every 6 hr PRN for fever. It is available as an oral solution of 20 mg/mL. How many milliliters (mL) of ibuprofen should be given to the client per dose? Record your answer using one decimal place. Answer: ___________ mL

a) Cleans the disposable stethoscope with chlorhexidine solution before resume with a different client Clients with a health care-associated infection, such as methicillin-resistant Staphylococcus aureus, are placed on contact precautions to prevent transmission of microorganisms. Contact precautions include standard precaution measures in addition to use of a gown and gloves and single-client-use equipment (eg, stethoscopes, blood pressure cuffs, thermometers). Disposable or single-client-use equipment must not be shared between clients or transferred to other care areas. Dedicated equipment should be kept in the room for client care, and then disinfected or discarded when no longer needed (option 1). (option 2) The urine specimen should be placed in a leak-proof specimen cup and then sealed in a biohazard bag before transport to the laboratory. (option 3) To prevent specimen contamination and the introduction of bacteria into the client's urinary tract, the nurse should scrub the Foley collection port with alcohol or chlorhexidine for 15 seconds before withdrawing a specimen. (option 4) Hand hygiene with an alcohol-based hand rub is recommended, unless there is visible soiling of the hands with body fluids, or after contact with Clostridium difficile. In both situations, hand hygiene must be performed with soap and water to thoroughly remove contaminants left behind by alcohol-based rubs. Educational objective: Nurses should implement contact precautions (eg, gown/gloves, single-client-use equipment) for clients with methicillin-resistant Staphylococcus aureus to prevent transmission of microorganisms. Single-client-use or disposable equipment should not be shared between clients. Hand hygiene with alcohol-based hand rubs is appropriate unless visible soiling or exposure to Clostridium difficile occurs.

A student nurse assesses and obtains a urine specimen form a client with methicillin-resistant Staphylococcus aureus (MRSA) who is on contact precautions. The registered nurse intervenes when the student performs which action? a) Cleans the disposable stethoscope with chlorhexidine solution before resume with a different client b) Removes the urine specimen cup from the room in a sealed biohazard bag c) Scrubs the Foley catheter collection port with alcohol for 15 seconds before withdrawing a urine specimen d) Uses an alcohol-based hand antiseptic solution after removing gloves

d) Pours sterile normal saline solution (NSS) into a sterile basin from a bottle opened 30 hours ago The sterility of an opened bottle of sterile saline cannot be guaranteed. Some institutions' policies permit recapped bottles of solution to be reused within 24 hours of opening, and some require disposal of the remaining solution. Therefore, the nurse should intervene when the student uses sterile saline from a bottle that was opened >24 hours ago. The general steps for preparing the sterile field for a wet-to-damp dressing change include: 1. Perform hand hygiene 2. Open a sterile gauze package that has a partially sealed edge with ungloved hands by grasping both sides of the edge, one with each hand, and pull them apart while being careful not to contaminate the gauze (option 2). 3. Hold the inverted opened gauze package 6" (15 cm) above the waterproof sterile field so it does not touch the field, and then drop the gauze dressing onto the sterile field (option 1). 4. Place the sterile dressings on the sterile field 2" (5 cm) from the edge; the 1" (2.5 cm) margin at each edge is considered unsterile because it is in contact with unsterile surfaces (option 3). 5. Use sterile NSS from a recapped bottle that was opened <24 hours ago (if policy permits). Educational objective: The general steps for preparing the sterile field for a wet-to-damp dressing change include: 1. Perform hand hygiene 2. Open a sterile gauze package with ungloved hands 3. Hold the inverted opened gauze package 6" (15 cm) above the sterile field 4. Place the sterile gauze dressing more than 1" (2.5 cm) from the edge of the sterile field. 5. Use sterile NSS from a recapped bottle that was opened <24 hours ago (if policy permits)

A student nurse prepares to change a large wet-to-damp sterile wound dressing and uses a disposable moisture-proof sterile drape to set up the sterile field. The perceptions nurse intervenes when the student performs which action? a) Holds the package 6" (15 cm) above the sterile field and drops the sterile gauze onto the field b) Opens the sterile gauze package with ungloved hands c) Places the sterile gauze dressings within 2" (5 cm) from the edge of the sterile drape d) Pours sterile normal saline solution (NSS) into a sterile basin from a bottle opened 30 hours ago

a) 38-year-old with methicillin-resistant Staphylococcus aureus b) 42-year-old with Clostridium difficile diarrhea d) 72-year-old with vancomycin-resistant Enterococcus Infections caused by methicillin-resistant Staphylococcus aureus (MRSA), C. difficile, vancomycin-resistant Enterococcus (VRE), and scabies require contact precautions to be used. Contact precautions include: -Placing client in private room (preferred) or cohorting clients with the same infection -Using dedicated equipment (must be disinfected when removing from room) - Wearing gloved when entering room - Perform proper hand hygiene before exiting room (use soap and water or alcohol-based hand rubs for MRSA and VRE, but only soap and water for C. difficile and scabies) - Wearing gown with client contact and removing before leaving room - Place door notice for visitors - Having client leave room only for essential clinical reasons (ie, tests, procedures). If an x-ray is needed, try to arrange for a portable one. (option 3) Clients with pertussis infection (whooping cough) need droplet precautions. (option 5) Influenza requires droplet precautions. Educational objective: Clients with multidrug-resistant organisms (MRSA, VRE), C. difficile diarrhea, and scabies require nursing staff to implement contact precautions.

The charge nurse is planning assignments for the day. Which clients will require the nursing staff to institute contact precautions? Select all that apply. a) 38-year-old with methicillin-resistant Staphylococcus aureus b) 42-year-old with Clostridium difficile diarrhea c) 69-year-old with pertussis infection d) 72-year-old with vancomycin-resistant Enterococcus e) 80-year-old with influenza

d) Client with pneumonia who has a positive methicillin-resistant Staphylococcus aureus nose culture A client with a positive nose was for methicillin-resistant Staphylococcus aureus (MRSA) is colonized and can transmit bacteria to others. If signs of infection are absent, treatment is not required. Colonized clients are at increased risk for infection with MRSA; if signs (eg, fever, wound drainage, purulent mucus) are present, treatment is required. The Centers for Disease Control and Prevention (CDC) recommends placing a colonized client on contact precautions and in a private room. The CDC also recommends that the highest priority be given to placing a colonized client who may transmit the bacteria through body secretions or excretions (eg, sputum, wound drainage) in a private room. Therefore, the client with pneumonia should be places in the private room. (options 1 & 3) The CDC recommends standard precautions for clients with hepatitis C and those who are HIV positive. A private room is not necessary for a client who has osteomyelitis or diabetic ketoacidosis. (Option 2) A client with a latent tuberculosis infection (LTBI) has a positive tuberculin skin test, has no symptoms of infection, and is not contagious. Immunosuppressant drugs, chemotherapy, and debilitating disease can convert LTBI to active disease. At this time, the client requires only standard precautions. Educational objective: The Centers for Disease Control and Prevention recommends contact precautions and private room placement for a client who is colonized with methicillin-resistant Staphylococcus areas, especially if the client can transmit the bacteria through body secretions or excretions.

The charge nurse must assign rooms to 4 clients who are scheduled for admission. Which client has the highest priority for a private room assignment? a) Client who is a known IV drug abuser who has osteomyelitis of the arm and chronic hepatitis C b) Client with chronic obstructive pulmonary disease who has a latent tuberculosis infection c) Client with diabetes mellitus and HIV infection who is in diabetic ketoacidosis d) Client with pneumonia who has a positive methicillin-resistant Staphylococcus aureus nose culture

a) Apply color-coded, nonslip socks to the client's feet b) Move the client to a room closer to the nurses' station c) Place a bedside commode to the right of the client e) Use a bed alarm to alert staff when the client gets up The client with right-brain damage following a stroke often experiences left-sided weakness, spatial-perceptual deficits, and impulsiveness, making this client at high risk for falls. Other factors that increase fall risk for older adults include: - Unfamiliar surroundings - Unsteady gait, decreased strength and coordination - Altered mental status - Orthostatic hypotension (related to dehydration) - Bowel/bladder urgency and/or frequency Application of color-coded, nonslip socks helps prevent a client from slipping and alerts staff to a client's increased risk for falls (option 1). Placing a commode by the right (stronger) side of the bed decreases the number of steps and time needed to get to a toilet (option 3). It also decreases the chance of tripping on equipment (eg, IV pump, tubing). Moving the client to a room close to the nurses' station allows frequent observation and a faster response time to calls for assistance (option 2). A bed alarm alerts staff when the client attempts to get out of bed, which allows for prompt response (option 5). (option 4) Raising all bed rails may be constituted as an unlawful use of restraint. Clients with altered mental status may also attempt to climb the side rails and sustain a fall injury. Educational objective: Many falls are associated with bathroom urgency/frequency. Fall risk precautions include placing the client in a room near the nurses' station, placing a bedside commode by the client's stronger side, applying nonslip socks, and using a bed alarm.

The nurse admits an 80-year-old client with an altered level of consciousness and left-sided weakness following a recent stroke. The client is dehydrated from multiple episodes of diarrhea. Which interventions should the nurse implement to prevent falls? Select all that apply. a) Apply color-coded, nonslip socks to the client's feet b) Move the client to a room closer to the nurses' station c) Place a bedside commode to the right of the client d) Raise all bed rails before leaving the room e) Use a bed alarm to alert staff when the client gets up

890 mL Peritoneal dialysis allows waste products to be removed from the bloodstream through semipermeable membrane of the peritoneum. Dialysate (ie, dialysis fluid) is infused into the peritoneal cavity, retained for a prescribed dwell time (eg, 20 minutes), and then drained as dialysate outflow. For clients on peritoneal dialysis, fluid balance should be tracked closely with daily weights and strict intake and output monitoring. Net fluid balance is calculated by subtracting total output from total intake. Educational objective: For clients on peritoneal dialysis, fluid balance should be tracked closely with daily weights and strict intake and output monitoring. Net fluid balance is calculated by subtracting total output from total intake. I

The nurse cares for a client receiving intermittent peritoneal dialysis who is prescribed strict intake and output monitoring with calculation of net fluid balance each shift. Calculate the total net fluid balance for the shift. Record the answer using a whole number. Click the exhibit button for additional information. Parenteral intake: 150 mL vancomycin IV (0700) 100 mL cefepime IV (1300) Oral intake: 240 mL coffee (0900) 120 mL tea (1100) 180 mL juice (1500) Other intake: 1500 mL dialysate (0900) Output: 1400 mL dialysate outflow (1300) Answer: ________________

b) Stop infusing the solution for 30 seconds, then resume at a slower rate Too rapid infusion of an enema solution may cause intestinal spasms that result in a feeling of fullness, cramping, and pain. If the client responds any of these symptoms, instillation should be stopped for 30 seconds and then resumed at a slower rate. Slow infusion will also decrease the likelihood of premature ejaculation of the solution, which would not allow for adequate bowel evacuation. (option 1) Having the client take slow, deep breaths may be helpful, but the infusion should be stopped first. (option 3) This response disregards the client's cramping and pain and is not appropriate. (option 4) Withdrawing the tube will risk not instilling the fluid high enough into the rectum/colon to be effective Educational objective: If a client reports cramping or pain during instillation of an enema, the infusion should be stopped for 30 seconds and then resumed at a slower rate.

The nurse is administering cleansing enemas to a client the night before bowel surgery. During instillation of the enema, the client reports cramping and pain. What action should the nurse take? a) Have the client take slow, deep breaths b) Stop infusing the solution for 30 seconds, then resume at a slower rate c) Tell the client that the process will not take much longer d) Withdraw the tube approximately 2 cm and continue the infusion

b) Deflate the balloon on the urinary catheter Because signs of traumatic injury are present, the nurse should follow steps to remove the catheter before further complication such as obstruction occur. Steps for removing an indwelling catheter include: - Perform hand hygiene - Ensure privacy and explain the procedure to the client - Apply clean gloves - Place a waterproof pad underneath the client - Remove any adhesive tape or device anchoring the catheter - Follow specific manufacturer instructions for balloon deflation - Loosen the syringe plunger and connect the empty syringe hub into the inflation port - Deflate the balloon by allowing water to flow back into the syringe naturally, removing all 10 mL, or applicable amount (note the size of the balloon labeled on the balloon port). If water does not flow back naturally, use only gently aspiration. - Remove the catheter gently and slowly; inspect to make sure it is intact and fragments were not left in the client. - If any resistance is met, stop the removal procedure and consult with the urologist for removal - Empty and measure urine before discarding the catheter and drainage bag in the biohazard bin or according to hospital policy - Remove gloves and perform hand hygiene (option 1) A urine specimen can be collected after the balloon is deflated or after the catheter is removed if needed. (option 4) The meatus should be cleaned after balloon deflation. Educational objective: When the urinary catheter balloon occluded the urethra, it should be deflated immediately to prevent further injury or complication. After balloon deflation, gently and slowly remove the catheter. If there is resistance, notify the urologist.

The nurse is assigned to care for a hospitalized confused client with an indwelling urinary catheter. On entering the client's room, the nurse notes the client pulling at the catheter and grimacing in pain. Blood is trickling from the client's meatus and the urine in the drainage bag is pink. Which action should the nurse take first? a) Collect a urine specimen and send to the lab b) Deflate the balloon on the urinary catheter c) Remove the catheter by gently pulling from the urethra d) Use a sterile 4x4 pad to absorb the blood around the meatus

a) Assess for rising pulse and respiration's afterward b) Check PT/INR and PTT values before the procedure c) Ensure that the client's blood is typed and crossmatched The client's coagulation status is checked before the liver biopsy using PT/INR and PTT. The liver ordinarily produces many coagulation factors and is a highly vascular organ. Therefore, bleeding risk should be assessed and corrected prior to the biopsy (option 2). Blood should be typed and crossmatched in case hemorrhage occurs (option 3). After the procedure, frequent vital sign monitoring is indicated as the early signs of hemorrhage are rising pulse and respirations, with hypotension occurring later (option 1). (option 4) The needle is inserted between ribs 6 and 7 or 8 and 9 while the client lies supine with the right arm over the head and holding the breath. A full bladder is a concern with paracentesis when a trocar needle is inserted is inserted into the abdomen to drain ascites. An empty bladder may air comfort, but it is not essential for safety. (option 5) The client must lie on the right side for a minimum of 2-4 hours to splint the incision site. The liver is a "heavy"organ and can "fall on itself" to tamponade any bleeding. The client stays on bed rest for 12-14 hours. Educational objective: Essential nursing actions related to a needle liver biopsy include checking coagulation, blood type, and cross match beforehand, positioning the client on the right side for hours afterward, and monitoring vital signs and for potential signs of shock.

The nurse is assisting a client who has a bedside needle liver biopsy scheduled. Which are the essential actions? Select all that apply. a) Assess for rising pulse and respiration's afterward b) Check PT/INR and PTT values before the procedure c) Ensure that the client's blood is typed and crossmatched d) Have the client void to ensure an empty bladder e) Position the client flat or on the left side after the procedure

b) 45-year-old with acute gastroenteritis and dehydration Acute gastroenteritis is associated with nausea, vomiting, and diarrhea, placing the client at risk for dehydration and sodium loss. Clients with gastroenteritis are encouraged to increase fluid intake but may require IV fluid therapy. Isotonic crystalloid fluids (eg, 0.9% sodium chloride, lactated Ringer solution) are the treatment of choice due to similarity in concentration with plasma and ability to increase extra cellular fluid (ECF) without moving into the intracellular space. In addition, isotonic fluids may increase sodium levels in clients experiencing excess sodium loss (eg, vomiting, diarrhea) (option 2). (option 1) Hypertonic solutions (eg, 3% sodium chloride) are administered to clients with increased intracranial pressure (ICP) to raise the osmolality of ECF, which decreases cellular swelling by drawing water from the cells. Isotonic fluids are avoided because an increase in circulating isotonic fluid promotes additional fluid shifting into the cells, which further increases ICP. (option 3) The client with a serum sodium of 112 mEq/L (112 mmol/L) is dangerously hyponatremic and at risk for further neurological decline. A hypertonic solution (eg, 3% sodium chloride) is the most appropriate choice to rapidly correct sodium deficits. (option 4) Isotonic solutions can exacerbate fluid overload in clients with chronic renal failure and cause increased blood pressure. Educational objective: Isotonic fluid therapy is used to treat clients with extra cellular fluid deficits (eg, dehydration). Clients at risk for cerebral swelling (eg, increased intracranial pressure, hyponatremia) require hypertonic fluid administration to decrease cellular swelling. Isotonic fluid administration may cause fluid overload in clients with renal failure.

The nurse is caring for 4 clients requiring IV fluid therapy. For which client should the nurse anticipate the need for isotonic crystalloid administration? a) 25-year-old with a closed-head injury and signs of increasing intracranial pressure b) 45-year-old with acute gastroenteritis and dehydration c) 60-year-old with seizures and serum sodium of 112 mEq/L (112 mmol/L) d) 68-year-old with chronic renal failure and hypertensive crisis

a) Have the client wear a mask Droplet precautions are used to prevent transmission of respiratory infection. These precautions include the use of a mask and a private room. When the client is in the room, staff should wear masks and follow standard precautions. The client on droplet precautions should wear a mask at all times when outside the hospital room. (option 2) Gloves are not required as part of droplet precautions. Standard precautions should guide the use of gloves in clients on droplet precautions. (option 3) The transporter does not need to wear a mask outside of the client's room as long as the client keeps a mask on to prevent transmission of infection. (option 4) An isolation gown is not required for droplet precautions. Educational objective: Droplet precautions require the use of regular masks to prevent the transmission of infection. A mask should be worn by the client when outside the hospital room and by staff when in the client's room.

The nurse is caring for a client on droplet precautions who has a prescription for a CT scan. When transporting the client to radiology, the nurse should ensure that the transporter uses protective equipment correctly to reduce the environmental spread of infection when the client is outside the room. Which instruction should the nurse give the transporter? a) Have the client wear a mask b) Have the client wear gloves c) Wear a mask d) Wear an isolation gown

a) Level of alertness b) Lung sounds c) Oxygen saturation d) Respiratory pattern Thoracentesis is commonly used to treat pleural effusion. The health care provider (HCP) will prepare the skin, inject a local anesthetic, and then insert a needle between the ribs into the pleural space where the fluid is located. A complication of thoracentesis is pneumothorax, which occurs when the needle goes into the lung and causes the lung to slowly deflate, like a balloon with a small hole in it. Bleeding is another, yet less common, complication of the procedure. Signs of pneumothorax include increased respiratory rate, increased respiratory effort, respiratory distress, low oxygen saturation, and absent breath sounds on the side where the procedure was done (where the lung is collapsed) (options 2,3, and 4). Tension pneumothorax may also develop, with tracheal shift to the unaffected side, severe respiratory distress, and cardiovascular compromise. Altered level of consciousness may occur due to decreased oxygenation and blood flow to the brain (option 1). A tension pneumothorax may be prevented by early detection of pneumothorax through appropriate monitoring. (option 5) Infection would be a later complication (occurring a few days after the procedure), so monitoring temperature is not required during the initial post procedure period. (option 6) Urine output should not be affected by thoracentesis or the drugs administered for this procedure. Educational objective: Following thoracentesis, the nurse should monitor for signs of pneumothorax, including level of alertness, respiratory rate, respiratory effort, oxygen saturation, and lung sounds

The nurse is caring for a client who is having a thoracentesis. Following the procedure, the nurse monitors for complications. The initial post procedure monitoring plan should include what? Select all that apply. a) Level of alertness b) Lung sounds c) Oxygen saturation d) Respiratory pattern e) Temperature f) Urine output

Answer: 50 gtt/min Educational objective: To calculate the drip rate of ampicillin, the nurse should identify the prescribed dose (eg, 1.5 g/dose) and available medication (eg, 1.5 g/100mL) and then convert to the rate in drips per minute (50 gtt/min).

The nurse is caring for a client who is prescribed ampicillin 1.5 g in 100 mL of normal saline IV to be administered over 30 minutes every 6 hours. The nurse has IV tubing with a drip factor of 15 gtt/mL. At what rate in drips per minute (gtt/min) should the nurse administer the IV ampicillin? Record your answer using a whole number. Answer: ______________________

c) Handwashing d) N95 particulate respirator Isolation is mandatory for clients with conditions that involve airborne transmission, and rooms must use both negative air pressurization and high-efficiency particulate air (HEPA) filters to avoid contamination. A class N95 or higher particulate respirator must be worn during client care. All clients with symptoms consistent with a suspected airborne illness should be given a surgical mask to wear as soon as they are assessed during triage. Gods hand hygiene is always the first and last element of infection control in any client care setting. (options 1 and 2) Wearing a gown and face shield would be necessary only if the nurse suspected splash of body fluids from procedural client care, not from routine care such as assessment or medication administration. Contact precautions may also be necessary if the tuberculosis is extra pulmonary with draining lesions (eg, cutaneous tuberculosis). (option 5) For client care involving airborne precautions, a class N95 or higher respirator must be used in lieu of a surgical mask to avoid potential exposure to aerosolized particles. Surgical masks are rated for barrier protection for droplet splashing and filtration of large respiratory particles only. Clients should be given surgical masks during their transportation. Educational objective: Tuberculosis requires airborne precautions. Clients suspected of having tuberculosis should be given a surgical mask to wear on entering any health care setting. Clients are placed in negative-pressure isolation rooms. Nurses must use a class N95 or higher particulate respirator.

The nurse is caring for a client with active pulmonary tuberculosis. Which elements of infectious disease precautions are mandatory for the nurse when providing routine care? Select all that apply. a) Gown b) Goggles or face shield c) Handwashing d) N95 particulate respirator e) Surgical mask

Answer:

The nurse is caring for a client with gastroenteritis and dehydration who is prescribed strict intake and output monitoring with calculation of net fluid balance each shift. Calculate the client's net fluid balance for the shift. Record your answer as a whole number. Click the exhibit button for additional information.

c) Release restraints at regular intervals and assess behavior e) Use gauze to pad bony prominences under restraints When caring for a client in restraints, the nurse should implement those interventions at regular intervals, according to agency policy (eg, every 2 hours): - Provide skin care and range-of-motion exercises; ensure basic needs are met (eg, fluids, nutrition, elimination). - Assess skin integrity and neurovascular status of restrained extremities; pad bony prominences under restraints, if necessary, to protect skin (option 5). - Determine the need for continued restraint by releasing restraints briefly and assessing the client's reaction; regularly assessing the need for restraints promotes discontinuation as soon as possible (option 3). (option 1) Restraint straps should be attached to areas that move with the bed frame (ie, elevates with the frame and head of the bed). Areas that do not move with (eg, base) or move independently of (eg, side rails) the frame should never be used, as injury may occur when they are raised or lowered (eg, pulling, entrapment). (option 4) Restraint straps should be tied in a quick-release knot, in case of emergency, and never in a square knot, which is difficult to release quickly. Educational objective: Nurses caring for restrained clients must ensure that basic needs are met, assess skin integrity and neurovascular status of restrained extremities, and determine the need for continued use. Supine position is avoided to decrease aspiration risk. Quick-release knots are used to attach restraints to parts of the bed frame that move with bed position changes.

The nurse is caring for an agitated client with dementia who is pulling at the oxygen and IV tubing. Wrist restraints are applied after less-restrictive safety measures have been ineffective. Which actions are appropriate to protect the client form injury? Select all that apply. a) Attach wrist restraint straps to the upper side rails b) Position the client supine to keep restraint straps taut c) Release restraints at regular intervals and assess behavior d) Use a square knot to tie restraint straps to the bed e) Use gauze to pad bony prominences under restraints

d) Return the client to a recumbent position and notify the health care provider Orthostatic vital signs help assess the body's ability to compensate hemodynamically during postural changes. Changing position normally triggers vasoconstriction in the extremities to promote venous return. Without this response, hypotension and subsequent hypoperfusion of internal organs and the brain occur. Clients with impaired compensatory mechanisms (eg, hypovolemia, sepsis) may exhibit orthostatic hypotension, in which hypotension and/or neurologic impairment (eg, syncope) occur with position change. This increases the client's risk for falls. Orthostatic vital signs involve measuring the client's cloud pressure (BP) and heart rate in the supine, sitting, and standing positions. Each measurement should be obtained after maintaining each position for 2 minutes. If any position change produces decreased systolic BP >20 mmHg, decreased diastolic BP > 10 mmHg, and/or increased pulse >20/min from supine values, the nurse should discontinue assessment, place the client in a recumbent position, and notify the health care provider (option 4). (option 1) It is unsafe to assist the client to a standing position after identifying orthostatic hypotension, as a syncopal event may occur and the client may fall. (options 2 and 3) Positioning the client in reverse Trendelenburg position and reassessing BP at a different site in the supine position are unnecessary and delay treatment of orthostatic hypotension. Educational objective: Nurses measuring orthostatic vital signs should discontinue further measurements if the client experiences decreased systolic blood pressure >20 mmHg, decreased diastolic blood pressure >10 mmHd, and/or increased pulse >20/min in any position. Return the client to a recumbent position and notify the health care provider.

The nurse is obtaining orthostatic vital signs on a client admitted for dehydration. The nurse measures the client's blood pressure and pulse using the left brachial site with the client lying supine and then sitting. Which action by the nurse is appropriate? Click on the exhibit button for additional information. Blood Pressure Supine: 153/83 mmHg Heart Rate Supine: 70/min Blood Pressure Sitting: 119/70 mmHg Heart Rate Sitting: 95/min a) Assist the client to a standing position and measure a third set of vital signs b) Place the client in reverse Trendelenburg position and take an apical pulse c) Reassesses the client's blood pressure in the supine position using the popliteal site d) Return the client to a recumbent position and notify the health care provider

Correct response: b) Clamp the catheter tubing d) Place the client in Trendelenburg position on the left side a) Administer oxygen as needed c) Notify the health care provider (HCP) e) Stay with the client and provide reassurance Leakage of more than 500 mL of air into a central venous catheter is potentially fatal. An air embolism in the small pulmonary capillaries obstructs blood circulation. A central venous catheter leaks air rapidly at 100 mL/sec. This client requires immediate intervention to prevent further complications (eg, cardiac arrest, death). The nurse should not delay emergency treatment, not even to stop and contact the HCP or the rapid response team (RRT). Priority interventions for active or suspected air embolism are as follows: 1. Clamp the catheter to prevent more air from embolizing into the venous circulation. 2. Place the client in Trendelenburg position on the left side, causing any existing air to rise and become trapped in the right atrium 3. Administer oxygen if necessary to relieve dyspnea. 4. Notify the HCP or call an RRT to provide further resuscitation measures 5. Stay with the client to provide reassurance and monitoring as the air trapped in the right atrium is slowly absorbed into the bloodstream over the course of a few hours. Educational objective: Any delay in treatment of an air embolism could prove fatal. There is no time to call the HCP. Seal off the source of the leak, and ensure stabilization of the air bubble via left lateral positioning.

The nurse is performing a central line tubing change when the client suddenly begins gasping for air and writhing. Order the interventions by priority. All options must be used. Unordered options: a) Administer oxygen as needed b) Clamp the catheter tubing c) Notify the health care provider (HCP) d) Place the client in Trendelenburg position on the left side e) Stay with the client and provide reassurance

b) Breaks the ampule neck away from the nurse's body to prevent injury from the glass c) Disposes of the empty glass ampule in a sharps container A glass ampule is a single-dose medication container with a scored area on the neck that must be broken to withdraw the medication. When preparing medication from a glass ampule, the nurse ensures safety and prevents contamination during medication administration by: - Flicking the upper stem of the ampule with a fingernail several times to ensure removal of medication from the ampule neck - Using sterile gauze to break the ampule neck away from the nurse's body to prevent injury from glass shards (option 2) - Setting the ampule on a flat surface or inverting it to withdraw the medication - Disposing of the ampule in a sharps container (option 3) (option 1) Glass shards may be present in the medication after an ampule is opened. To prevent the accidents; administration of glass shards, the nurse must use a filter needle, rather than an injection needle, when withdrawing medication. (option 4) Unlike when withdrawing medication from a vial, air should not be injected into a glass ampule; this causes the contents to spill from the container. (option 5) Ensure that the filter needle does not touch the glass edges, which are not sterile, as this can introduce bacteria. Educational objective: When preparing medication from a glass ampule, the nurse breaks the ampule away from the body and discards it in the sharps container. The nurse withdraws medication using a filter needle to prevent the injection of glass shards, avoids touching the needle to contaminated ampule edges, and avoids injecting air to prevent spillage.

The nurse is preparing an injection of IM haloperidol from a glass ampule. Which of the following actions by the nurse are appropriate? Select all that apply. a) Attaches an 18 gauge injection needle to a syringe for withdrawal of medication b) Breaks the ampule neck away from the nurse's body to prevent injury from the glass c) Disposes of the empty glass ampule in a sharps container d) Injects air into the glass ampule prior to withdrawing the medication e) Rests and steadies the needle on the ampule's outer rim to withdraw medication

c) Discontinue the D5W, flush the IV catheter with normal saline, and start the transfusion Normal saline (NS) is the only fluid that can be given with a blood transfusion. Dextrose solutions may lyse the red blood cells. All other IV solutions and medications may cause precipitation and are incompatible with blood. Blood transfusions should be infused through a dedicated IV line. If a transfusion must be started in an IV catheter currently in use, the nurse should discontinue the infusion(s) and tubing, and then flush the catheter with NS prior to connecting the blood administration tubing. After the transfusion, the catheter should be cleared with NS before any other IV fluids are administered. (option 1) Packed red blood cells are not compatible with D5W. The nurse must discontinue the D5W and flush the catheter with NS before administering blood. (option 2) Although an 18-gauge IV catheter is preferred for blood administration, a 20-gauge catheter is acceptable. The nurse can start a second IV catheter if required, but there is no need to discontinue the original one. (option 4) Blood should not be run with any other fluid except NS. Blood can be infused with an IV pump if the fluid in the tubing is compatible. Educational objective: Blood transfusions cannot be run with any other IV fluid except normal saline (NS). Dextrose can lyse the red blood cells, and other fluids can cause precipitation. If another fluid has been infused through an IV catheter, the nurse should discontinue the infusion(s) and tubing and flush with NS before administering blood.

The nurse is preparing to administer a unit of packed red blood cells to a 16-year-old with blood loss anemia. The client currently has D5W infusing through a 20-gauge IV catheter. What action should the nurse take? a) Attach the blood transfusion set to the port closest to the client on the existing IV tubing b) Discontinue the 20-gauge IV catheter and restart an 18-gauge IV catheter c) Discontinue the D5W, flush the IV catheter with normal saline, and start the transfusion d) Run the blood transfusion as an IB piggyback through the infusion pump

c) 1-in (25-mm) needle in the anterolateral thigh The needle length and injection site for IM injections are dependent on a client's age and muscle mass. The vastus lateralis muscle in the anterolateral middle portion of the thigh is the preferred IM injection site for newborns (age <1 month) and infants (age 1-12 months). Selection of the most appropriate needle length is an important factor in ensuring immunization success and minimizing local reactions to vaccine components. If the needle is too short, the IM vaccine is injected into subcutaneous fat, resulting in vaccine failure due to poor mobilization of the antigen within adipose tissue. Infants typically require a 1-in (25-mm) needle for IM injections (option 3). (option 1) A 3/8-in (9-mm) needle is too short to penetrate deep vastus lateralis muscle of the thigh (options 2 and 4) The ventrogluteal area in an infant does not have enough muscle mass for use and is not recommended until at least age 3. A 5/8-in (16-mm) needle is too short and does not penetrate the deep muscle. A 1.5-in (38 mm) length is too long and is normally used on older children and adults with sufficient muscle mass. Educational objective: Appropriate needle length and injection site are essential for proper administration of IM immunizations. For infants, use a 1-in (25-mm) needle to administer medication in the vastus lateralis muscle is recommended to reach the IM tissue and minimize local reactions.

The nurse is preparing to administer an IM immunization to a 6-month-old infant. Which needle length and injection site would be the most appropriate to minimize a local reaction to the vaccine components? a) 3/8-in (9-mm) needle in the anterolateral thigh b) 5/8-in (16-mm) needle in the ventrogluteal muscle c) 1-in (25-mm) needle in the anterolateral thigh d) 1.5-in (38-mm) needle in the ventrogluteal muscle

b) Crush each medication separately before administration c) Determine if the medications are available in liquid form d) Flush the tube before and after medication administration Failure to correctly administer medications through feeding tubes (eg, nasogastric, gastrostomy) can result in obstruction of the tube, reduced medication absorption or efficacy, and medication toxicity. Before administering medications through a feeding tube, the nurse should determine if any of the medications are available in a liquid form because liquid medications are less likely to clog the tube (option 3). Medications should be crushed, dissolved, and administered separately to prevent interactions (eg, chemical reactions) between medications or interference with absorption (option 2). In addition, a feeding tube should be flushed before and after each medication is given to avoid potential drug interactions to ensure tube patency (option 4). (option 1) When using a feeding tube, each medication should be administered individually to prevent interactions between medications. (option 5) Medications mixed with enteral feedings may form a thick consistency and clog the tube. Educational objective: When using a feeding tube, medications should be crushed, dissolved, and administered separately to prevent interactions Feeding tubes should be flushed before and after each medication is given. Liquid medications should be used if possible.

The nurse is preparing to administer several medications through a client's feeding tube. None of the medications are extended release. Which of the following actions should the nurse implement? Select all that apply. a) Combine all medications and administer together b) Crush each medication separately before administration c) Determine if the medications are available in liquid form d) Flush the tube before and after medication administration e) Mix medications with enteral feeding formula before administration

c) 10 mL Flushing the lumen of a central venous access device (central venous catheter [CVC]) with normal saline is recommended to assess patency before medication infusion, prevent medication incompatibilities after infusion, and prevent occlusion after blood sampling. A 10-mL syringe is generally preferred for flushing the lumen of a CVC (option 3). The smaller the syringe, the greater the amount of pressure per square inch exerted during injection, increasing the risk for damage to the CVC. The "push-pause" method involves slowly injecting normal saline into the CVC catheter and stopping for any resistance. Injecting against resistance can damage the CVC, which may result in complications, including embolism and malfunction. The nurse should always consult the specific manufacturer guidelines and facility policy when caring for a CVC. (options 1 and 2) A smaller syringe (eg, 1 mL, 3 mL) creates more pressure, which increase the risk for damage to the CVC. (option 4) A 30-mL syringe is unnecessarily large to flush a CVC Educational objective: When flushing the lumen of a central venous catheter, the nurse should use the safest syringe possible and the "push-pause" method to avoid exerting too much pressure, which may damage the catheter. The smaller the syringe, the greater the amount of pressure exerted during the flush. A 10-mL syringe is generally recommended; however, it is also important to consult the manufacture's guidelines.

The nurse is preparing to flush a client's central venous catheter. Which size syringe is best for the nurse to choose? a) 1 mL b) 3 mL c) 10 mL d) 30 mL

When administering subcutaneous anticoagulant injections (eg, heparin, enoxaparin), the nurse must select the appropriate needle length and angle to avoid accidental intramuscular injection, especially in clients with insufficient adipose tissue (eg, cachexia). Intramuscular injection of heparin would cause rapid absorption, resulting in a hematoma and painful muscle irritation. The nurse should administer subcutaneous injections at 90 degrees if 2 in (5 cm) of subcutaneous tissue can be grasped, or at 5 degrees if only 1 in (2.5 cm) can be grasped (option 2). Anticoagulants are best absorbed if administered in the abdomen at least 2 in (5 cm) away from the umbilicus. (option 1) A 15-degree angle is used for intradermal injections and would not deliver medication into the subcutaneous tissue. (option 3) A 90-degree injection angle is appropriate for clients with sufficient adipose tissue. (Ie, at least 2 in [5 cm] can be grasped). (option 4) Needles longer than 5/8 in (1.6 cm) are used to administer intramuscular injections. Educational objective: Anticoagulant injections should be administered in the abdominal subcutaneous tissue at a 45- to 90-degree angle. A 45-degree angle is used for clients with minimal adipose tissue to avoid accidental intramuscular injection, which would cause rapid absorption and result in hematoma and painful muscle irritation.

The nurse is preparing to give a heparin injection to a client who is malnourished and cachectic. Which method of injection would be appropriate for this client?

a) Assess client's vital signs c) Obtain a Y tubing set and prime with normal saline (NS) d) Plan to remain with the client during the 1st 15 minutes of transfusion The procedure for safe blood administration includes the following: 1. Obtain a unit of blood from the blood bank and verify the blood product with type and cross match results and at least 2 client identifiers with another nurse at the client's bedside. The blood is obtained and infused one unit at a time (option 2). 2. Assess the client, obtain vital signs for baseline, and teach signs of a transfusion reaction and how to call for help. 3. Use a Y tubing, prime with NS, and then clamp the NS side (option 6). 4. Spike the blood product, leaving the blood side of the Y tube open while keeping the saline side clamped for infusion. The saline is only used to prime the tubing and flush after the infusion. It does not infuse simultaneously. 5. Set the infusion pump to deliver blood over 2-4 hours as prescribed (option 5). Rapid infusion of the blood puts the client at greater risk for transfusion reaction and fluid volume overload. 6. Remain with the client for at least the 1st 15 minutes and watch for signs of blood transfusion reaction, including fever, chills, nausea, vomiting, pruritus, hypotension, decreased urine output, back pain, and dyspnea. Stop the transfusion immediately if a reaction occurs. The first 15 minutes of infusion should be slow to watch for these reactions. 7. Take another set of vital signs 15 minutes after infusion starts and continue in accordance with facility policy. Always take a final set of vital signs after the infusion is complete. 8. On completion of the blood transfusion, open the saline side clamp of the Y tubing to flush all blood in the tubing through with NS. 9. Return the blood bag with the attached set-up to the laboratory after completion or dispose of in accordance with hospital policy. Use new IV Y tubing set-up fort the second unit of blood. Educational objective: Always verify blood products, type and cross match results, and client identifiers with another nurse prior to infusion. Obtain vital signs before, during, and after blood administration. Watch for transfusion reaction and stop the transfusion immediately if a reaction occurs.

The nurse is preparing to infuse 2 units of packed red blood cells (PRBCs) to a client with a gastrointestinal bleed. Which actions should the nurse take? Select all that apply. a) Assess client's vital signs b) Infuse both units simultaneously c) Obtain a Y tubing set and prime with normal saline (NS) d) Plan to remain with the client during the 1st 15 minutes of transfusion e) Set infusion pump to deliver unit over 30 to 45 minutes f) Spike filtered intravenous (IV) tubing with dextrose 5% water (D5W)

f) Perform hand hygiene and don clean gloves e) Suction the oropharynx and perform oral care d) Hyperoxygenate the lungs (100% FiO2) a) Advance catheter into the trachea c) Gently rotate the catheter while suctioning b) Evaluate client tolerance and document Clients with endotracheal tubes (ETTs) have impaired cough and gag reflexes and require suction to clear retained bronchial secretions and promote ventilatory efficacy. Ventilator circuits for ETTs typically have a reusable in-line endotracheal suction device, which remains sterile, in a flexible plastic sleeve. Oral secretions may pool near the base of the ETT and drip into the trachea; therefore, oropharyngeal suctioning and oral care are performed before ETT suctioning to prevent introduction of oral bacteria into the lungs. The steps for suctioning an ETT include: 1. Perform hand hygiene and don clean gloves (option 5). 2. Suction the oropharynx and perform oral care (option 6). 3. Ensure that the system is connected to appropriate wall suction (<120 mmHg) 4. Hyperoxygenate the lungs (100% FiO2) (option 4). 5. Advance the catheter into the trachea just until resistance is met (level of the carina) (option 1). Do not suction while advancing the catheter. 6. Gently remove the catheter while suctioning and rotating it. Do not suction for more than 10 seconds (option 3). 7. Evaluate client tolerance; if further secretions remain, suctioning can be repeated 1 or 2 times. Document the procedure when complete (option 2). 8. Resume oxygenation and ventilation settings as prescribed. Educational objective: Tracheal suctioning through an endotracheal tube helps clear retained bronchial secretions and increases ventilatory efficacy. The nurse must follow strict asepsis to prevent the introduction bacteria into the lungs, hyperoxygenate the lungs to prevent hypoxia, and use appropriate technique to prevent airway trauma.

The nurse is preparing to suction secretions from the airway of an unconscious client whose lungs are mechanically ventilated with an endotracheal tube. Place the steps for suctioning the endotracheal tube in the correct order. All options must be used. Unordered options: a) Advance catheter into the trachea b) Evaluate client tolerance and document c) Gently rotate the catheter while suctioning d) Hyperoxygenate the lungs (100% FiO2) e) Suction the oropharynx and perform oral care f) Perform hand hygiene and don clean gloves

a) Apply suction for no longer than 5-10 seconds d) Wait at least 1 minute between suction passes The process of suctioning a client's airway removes oxygen in addition to the secretions; therefore, the client should be preoxygenated with 100% O2, and suction should be applied for no more than 10 seconds during each pass to prevent hypoxia (option 1). The nurse must wait 1-2 minutes between passes for the client to ventilate to prevent hypoxia (option 4). In addition, deep rebreathing should be encouraged. (option 2) The suction catheter should be no more than half the width of the artificial airway and inserted without suction. (option 3) The nurse should don sterile gloves if the client does not have a closed suction system in place. Suction should be set at medium pressure (100-120 mmHg for adults, 50-75 mmHg for children) as excess pressure will traumatize the mucosa and can cause hypoxia. (option 5) Clients usually cough as the catheter enters the trachea, and this helps loosen secretions. The catheter should be advanced until resistance is felt and then, to prevent mucosal damage, retracted 1 cm before applying suction Educational objective: Proper airway suctioning technique includes preoxygenation, limiting a suction pass to 10 seconds, and allowing 1-2 minutes between passes to prevent hypoxia. Medium suction pressure should be set at 100-120 mmHg for adults, with the catheter inserted without suction.

The nurse is suctioning the artificial airway of a conscious client. Which actions demonstrate correct technique? a) Apply suction for no longer than 5-10 seconds b) Insert catheter with low, intermittent suction applied c) Set suction higher than 130 mmHg for thick, copious secretions d) Wait at least 1 minute between suction passes e) Withdraw catheter immediately if client begins coughing

Correct response: a) Exhale normally and place the mouthpiece in the mouth e) Seal lips tightly on mouthpiece d) Inhale deeply, until piston is elevated to predetermined level c) Hold breath for at least 2-3 seconds b) Exhale slowly around the mouthpiece Incentive spirometer is recommended in postoperative clients to prevent atelectasis associated with incisional pain, especially in upper abdominal incisions (close to the diaphragm). Adequate pain medication should be administered before using the incentive spirometry. Guidelines recommend 5-10 breaths per session every hour while awake. Volume-oriented or flow-oriented sustained maximal inspiration (SMI) devices can be used. The client instructions for using a volume-oriented SMI device include: 1. Assume a sitting or high Fowler position, which optimizes lung expansion, and exhale normally 2. While holding the device at an even level, seal the lips tightly on the mouthpiece to prevent leakage of air around it 3. Inhale deeply through the mouth until the piston is elevated to the predetermined level of tidal volume. The piston is visible on the device and helps provide motivation. 4. Hold the breath for at least 2-3 seconds (up to 6 seconds) as this maintains maximal inhalation 5. Exhale slowly to prevent hyperventilation 6. Breathe normally for several breaths before repeating the process 7. Cough at the end of the session to help with secretion expectoration Educational objective: Incentive spirometry is recommended to prevent atelectasis in postoperative clients. Clients with incisional pain should receive adequate pain medication prior to the inhalations. The client is instructed to use the device while sitting upright, seal the lips tightly around the mouthpiece, inhale deeply, sustain maximal inspiration for at least 2-3 seconds, exhale slowly before repeating the procedure, and cough at the end of the session.

The nurse is teaching a postoperative client to use a volume-oriented incentive spirometer device. Place the teaching steps in the proper order. All options must be used. Unordered options: a) Exhale normally and place the mouthpiece in the mouth b) Exhale slowly around the mouthpiece c) Hold breath for at least 2-3 seconds d) Inhale deeply, until piston is elevated to predetermined level e) Seal lips tightly on mouthpiece

a) Descends with the cane on the steps first, followed by the left leg, and then the right leg To prevent falls after a total knee replacement, clients should use a cane to provide maximum support when climbing up and down any stairs. Clients should hold the cane on the stronger side and move the can before moving the weaker leg, regardless of the direction. Clients must also keep 2 points of support on the floor at all times (ie, both feet, foot and cane). When descending stairs, the client should: 1. Lead with the cane 2. Bring the weaker leg down next (in this client, it is the left leg) 3. Finally, step down with the stronger leg (option 1) When ascending stairs, the client should: 1. Step up with the stronger leg first 2. Move the cane next, while bearing weight on the stronger leg 3. Finally, move the weaker leg To remember the order, use the mnemonic "up with the good and down with the bad." The can always moves before the weaker leg. (options 2, 3, and 4) These options do not provide enough support to the weaker leg when descending. Educational objective: To prevent falls when descending the stairs using a cane, the client should lead with the cane, follow with the weaker leg, and then step down with the stronger leg.

The nurse observes a client who is postoperative left total knee replacement use a cane. Which action by the client indicates an understanding of the correct technique when walking down the stairs? a) Descends with the cane on the steps first, followed by the left leg, and then the right leg b) Descends with the cane on the step first, followed by the right leg, and then the left leg c) Descends with the left leg on the step first, followed by the cane, and then the right leg d) Descends with the right leg on the step first, followed by the left leg, and then the cane

a) Advances tip of needle through epidermis until the bevel is no longer visible under the skin Intradermal dermal injections deliver a small amount of medication (0.1 mL) into the dermal layer of the skin, just under the epidermis. This parenteral route is used to perform allergy testing and tuberculosis (TB) screening. The correct procedure for administering a TB intradermal injection is as follows: 1. Choose a 1 mL tuberculin syringe with a 27-gauge 1/4 inch needle then don clean gloves- the syringe is calibrated in hundredths of a millimeter and the intradermal needle is short enough to remain in the dermis with length range of 1/4-5/8 inch (option 2). 2. Position the left forearm to face upward, and cleanse site that is a hands width above the wrist- the left arm is commonly used for TB testing; the forearm has little hair and subcutaneous tissue and is readily accessible to observe a skin reaction. 3. Place non-dominant hand 1 inch below the insertion site and pull skin downward so that it is taut- taut skin makes it easier to insert the needle and promotes comfort. 4. Insert the needle almost parallel to skin at a 10-degree angle with bevel up- this is important as the medication can enter the subcutaneous tissue if the angle is > 15 degrees (option 4). 5. Advance the tip of the needle through epidermis into dermis; outline of bevel should be visible under the skin- verify that the medication will be injected into dermis (option 1) 6. Inject medication slowly while raising a small wheal (bleb) on the skin- verify that the medication is being deposited into the dermis (option 3) 7. Remove needle and do not rub the area- rubbing promotes leakage through the insertion site and medication deposition into the tissue. 8. Circle the area with a pen to assess for redness and induration (according to institution policy)- this delineates the border for measurement of reaction. Educational objective: For TB skin testing: 1. Use a 27-gauge 1/4 inch needle with a 1 mL tuberculin syringe 2. Administer injection on inner forearm at a 10-degree angle with bevel up 3. Make a wheal (bleb) 4. Avoid rubbing site after injection

The nurse observes a student administer a tuberculin skin test using the intradermal route. The nurse intervenes when the student performs which action? a) Advances tip of needle through epidermis until the bevel is no longer visible under the skin b) Chooses a 1 mL tuberculin syringe with a 27-gauge 1/4 inch needle; dons clean gloves c) Injects medication slowly while raising a small wheal (bleb) on the skin d) Inserts needle at a 10-degree angle almost parallel to skin with the bevel up

a) Femoral line inserted in emergency department post cardiac arrest 48 hours ago In adult clients, central venous access sites in the upper body (internal jugular or subclavian) are preferred to minimize the risk of infection. Access sites in the inguinal also area (femoral) are easily contaminated by urine or feces, and it is difficult to place an occlusive dressing over these sites. A central venous catheter (CVC) should be placed where aseptic technique can be applied. The site should be assessed daily for signs/symptoms of infection (eg, redness, swelling, drainage). The duration of CVC placement should be based on clinical need and judgement that there is no evidence of infection. (option 2) Although this site has been in use for 6 days, it is a preferred site; the CVC was inserted in the operating room, where surgical asepsis was easily accomplished. The site can be used as long as there is a clinical need and no evidence of infection. (option 3) Peripherally inserted central catheter (PICC) lines can be left in for weeks or months. The occlusion of one lumen does not necessitate removal of the catheter (option 4) The subclavian vein is a preferred site for a CVC. Although slight redness is present at the suture sites, it is not located at the insertion site. The femoral line is still at higher risk for infection. Educational objective: Femoral central venous catheter may be placed in emergency situations but should be removed/replaced as soon as possible due to the high risk of contamination and infection.

The nurse on the IV therapy team is making rounds in the intensive care unit on clients with central venous catheters. Which central line should be removed earliest to prevent infection? a) Femoral line inserted in emergency department post cardiac arrest 48 hours ago b) Internal jugular line inserted 6 days ago in operating room c) Peripherally inserted central catheter line with one lumen occluded that was placed 2 weeks ago d) Subclavian line with slight redness at anchor suture sites inserted in intensive care unit 72 hours ago

d) Pull back on the tube slightly and then pause to give the client time to breathe During NG tube insertion, the tube sometimes slips into the larynx or coils in the throat, which can result in coughing and gagging. The nurse should withdraw the tube slightly and then stop or pause while the client takes a few breaths. After the client stops coughing, the nurse can proceed with advancement (option 2), asking the client to take small sips of water to facilitate advancement to the stomach (option 1). The client should not be asked to swallow during coughing or aspiration may occur. If resistance or obstruction occurs during tube advancement, the nurse should rotate the tube while trying to advance it. If resistance continues, the tube should be withdrawn and inserted into the other naris if possible (option 3). Educational objective: Coughing and gagging commonly occur during NG tube insertion if the tube coils in the throat or slips into the larynx. When this happens, the nurse should pull back on the tube slightly and then pause to give the client time to recover and breathe before advancing the tube.

The nurse performs nasogastric (NG) tube insertion using a large-bore NG tube on a hospitalized client with a gastrointestinal bleed. During insertion, after the tube passes the nasopharynx, the client begins to cough and gag. Which action should the nurse take first? a) Ask the client to take several small sips of water b) Continue to slowly advance the tube until placement is reached c) Gently remove the tube and reinsert in the other naris if possible d) Pull back on the tube slightly and then pause to give the client time to breathe

b) 18-gauge When selecting catheter size, the need for rapid fluid administration and the type of fluid administered versus client discomfort should be assessed. A lower IV catheter gauge number corresponds to a larger bore IV catheter. 1. A 14-gauge (large-bore) catheter may be used for administering fluids and drugs in an emergency or prehospital setting, or for hypovolemic shock (option 1). 2. In somewhat stable adult clients who require large amounts of fluids or blood, an 18-gauge catheter is preferred. (options 3 and 4) A 20-22-gauge catheter is sufficient for administering general IV fluids and medications to adult clients; a 20-gauge is acceptable for blood transfusion. However, 20-22-gauge is not preferred for blood administration. A 24-gauge catheter is recommended for children and some older adults with small, fragile veins. Educational objective: A 14-gauge (large-bore) catheter is used to administer fluids and drugs in a prehospital or emergency setting, or for hypovolemic shock. An 18-gauge catheter is typically indicated for infusing blood or large amounts of fluid in adults.

The nurse plans to start an IV line to infuse 2 units of packed red blood cells for a stable 42-year-old client with a gastrointestinal bleed. Which IV catheter size is best? a) 14-gauge b) 18-gauge c) 20-gauge d) 22-gauge

d) Gown, gloves, N95 respirator, and eye protection Middle East respiratory syndrome (MERS) is a viral respiratory illness caused by the coronavirus (MERS-CoV). Symptoms include fever, cough, and shortness of breath that often worsen and cause death in many of those afflicted. The incubation period is 5-6 days but can range from 2-14 days. How the virus spreads is not fully understood, but it is though to spread via respiratory secretions. Because it has easily spread to those who care for infected persons, the Centers for Disease Control and Prevention recommends the use of standard, contact, and airborne precautions with eye protection when caring for clients with MERS. (options 1, 2, and 3) These options do not provide enough protection as each is missing a vital element that is recommended when caring for a client with MERS. Educational objective: Standard, contact, and airborne precautions with eye protection should be used when caring for a client with suspected or diagnosed Middle East respiratory syndrome.

The nurse prepares to care for a client being admitted with a confirmed diagnosis of Middle East respiratory syndrome. Which personal protective equipment will the nurse use when providing care to the client? a) Gloves and gown b) Gloves and mask c) Gown and N95 respirator d) Gown, gloves, N95 respirator, and eye protection

a) Clean the vial tops with alcohol swabs d) Inject air into the NPH insulin vial e) Inject air into the regular insulin vial c) Draw up the regular insulin solution b) Draw up the NPH insulin solution Mixing insulin allows multiple insulin preparations to be delivered in a single subcutaneous injection, thereby sparing the client from multiple injections. Intermediate-acting insulin's (eg, NPH) can be mixed with short-acting (eg, regular) or rapid-acting (eg, aspart, lispro) insulins. Most long-acting insulins (eg, glarine, determir) are not suitable for mixing and are typically packaged in prefilled syringes. When drawing up multiple insulin's, there is a risk for contaminating the shorter-acting vials with the longer-acting insulin, which would slow the action of later doses withdrawn from the shorter-acting insulin vial. Multi dose vials of regular insulin that have been contaminated with other insulins are unsafe for IV administration. When drawing up multiple insulins, the nurse should: 1. Clean both vial tops with alcohol swabs (option 1) 2. Inject air into the NPH insulin vial without touching the needle to the solution (option 4) 3. Withdraw the needle from the NPH insulin vial and inject air into the regular insulin vial (option 5). 4. Invert the regular vial and withdraw the regular solution into the syringe (option 3) 5. Insert the needle into the NPH insulin vial and withdraw the solution (option 2). The nurse can recall the pneumonic RN (Regular before NPH). Educational objective: When drawing up multiple insulins, there is a risk of contaminating the shorter-acting insulin vial with longer-acting insulin and slowing the action of later doses withdrawn from the shorter-acting insulin first, and then use the same syringe to withdraw the intermediate-acting insulin.

The nurse prepares to draw up regular and NPH insulin's into one syringe. Place in order the steps the nurse should take when mixing the insulin's. All options must be used. Unordered options: a) Clean the vial tops with alcohol swabs b) Draw up the NPH insulin solution c) Draw up the regular insulin solution d) Inject air into the NPH insulin vial e) Inject air into the regular insulin vial

f) Measure, mark, and lubricate tube e) Instruct client to extend neck back slightly d) Gently insert tube just past nasopharynx b) Ask the client to flex head forward and swallow a) Advance tube to the marked point c)Verify tube placement and anchor Steps for inserting a nasogastric tube for gastric decompression include the following: 1. Perform hand hygiene and apply clean gloves (no need for sterile gloves) 2. Place client in high Fowler's position 3. Assess nares and oral cavity and select naris 4. Measure and mark the tube 5. Curve 4-6" tube around index finger and release 6. Lubricate end of tube with water-soluble jelly 7. Instruct client to extend neck back slightly 8. Gently insert tube just past nasopharynx, aiming tip downward 9. Rotate tube slightly if resistance is met, allowing rest periods for client 10. Continue insertion until just above oropharynx 11. Ask client to flex head forward and swallow small sips of water (or dry if NPO) 12. Advance tube to marked point 13. Verify tube placement and anchor-use agency policy and procedure to verify placement by anchorIn tube in place and obtaining an abdominal x-ray. Aspirating gastric contents and testing the pH may also give an indication of placement (pH should be 5.5 or below). Auscultation of inserted air is acceptable for confirming tube placement initially, but is not definitive as it is not an evidence-based method. Nothing may be administered through the tube until x-ray confirmation is obtained, or this may cause aspiration. Educational objective: Key steps when inserting a large-bore nasogastric tube include using clean gloves; inspecting nares; measuring, marking, and lubricating tube; instructing client to extend the neck back slightly; inserting tube past the nasopharynx and continuing advancement until just above oropharynx; asking the client to flex the head forward and swallow; advancing tube to marked point; and verifying tube placement using abdominal x-ray anchoring.

The nurse prepares to insert a large-bore nasogastric tube for gastric decompression. After obtaining equipment, the nurse identifies the client, performs hand hygiene, applies clean gloves, assesses nares, and selects a naris. Place the remaining steps in the correct order. All options must be used. a) Advance tube to the marked point b) Ask the client to flex head forward and swallow c)Verify tube placement and anchor d) Gently insert tube just past nasopharynx e) Instruct client to extend neck back slightly f) Measure, mark, and lubricate tube

Correct response: b) Perform hand hygiene and open a sterile urinary catheterization kit a) Apply sterile gloves and place sterile drape under the client's buttocks f) Use the non dominant hand to gently spread the labial folds c) Use the dominant hand to cleanse the labial folds with antiseptic swabs d) Use the dominant hand to cleanse the urethral meatus with antiseptic swabs e) Use the dominant hand to insert the catheter until urine return is observed Steps for indwelling urinary catheter insertion for the female client include: - Position the client supine with knees flexed and hips slightly externally rotated - Perform hand hygiene and open a sterile catheterization kit (option 2) - Apply sterile gloved and place a sterile drape underneath the client's buttocks (option 1). - Remove the protective covering from the catheter, lubricate the catheter tip, and pour antiseptic solution over cotton balls or swab sticks while maintaining sterility of gloves and sterile field - Use the non dominant hand to gently spread the labia. The non dominant hand is now contaminated (option 6). - Use the dominant (sterile) hand to cleanse the labia and urinary meatus with antiseptic-soaked cotton balls or swab sticks. Cleanse in an anteroposterior direction (from the clitoris toward the anus). Use a new swab for each swipe to avoid transferring bacteria between areas. Cleanse the labia majora first, then the labia minors, and lastly the urinary meatus (options 3 and 4). - Use the dominant hand to insert the catheter until urine return is visualized in the tubing (usually 2-3 inch [5-7.6 cm]), and then advance it an additional 1-2 inch (2.5-5 cm) (option 5). - Hold the catheter in place with the non dominant hand, and then use the dominant hand to inflate the balloon. Educational objective: To insert an indwelling urinary catheter in a female client: perform hand hygiene; apply sterile gloves and place a sterile drape under the client; arrange supplies on a sterile field; gently spread the labia with the non dominant hand; cleanse the labia majora, then the labia minors, and lastly the urinary meatus; insert the catheter until urine return is visualized; advance an additional 1-2 inch (2.5-5 cm); and inflate the balloon.

The nurse prepares to insert an indwelling urinary catheter for a female client. The nurse assesses for allergies, explains the procedure to the client, gathers equipment, and then performs perineal care. Place in order the steps the nurse should take when inserting the urinary catheter. All options must be used. Unordered options: a) Apply sterile gloves and place sterile drape under the client's buttocks b) Perform hand hygiene and open a sterile urinary catheterization kit c) Use the dominant hand to cleanse the labial folds with antiseptic swabs d) Use the dominant hand to cleanse the urethral meatus with antiseptic swabs e) Use the dominant hand to insert the catheter until urine return is observed f) Use the non dominant hand to gently spread the labial folds

b) Maintain a sterile field and keep the urinary catheter sterile Maintaining sterility is key when inserting an indwelling urinary (Foley) catheter. Sterile technique (surgical asepsis) should be used to prevent catheter-associated urinary tract infections, the 2nd most common type of health care-associated infection. Proper placement of the kit and application of sterile gloves are important to maintaining sterility. The dominant hand should be kept sterile until the procedure is complete. Once the non dominant hand is placed on the client's genitalia (to spread the labia or grasp the penis) for cleansing, it should be kept in place until the catheter is inserted to prevent contamination. (option 1) The procedure should be explained to the client; however, this client is confused and likely will not understand. (option 3) The catheter kit may be placed between the client's legs if the client is cooperative. However, use of a clean bedside table is more effective in maintaining a sterile field when the client is confused and uncooperative. (option 4) Swabs should be disposed of in the trash can or biohazard bin in accordance with hospital policy; however, maintaining a sterile field is a higher priority. Educational objective: Maintenance of sterile technique is a key outcome for indwelling urinary catheterization. A sterile field should be maintained during the entire procedure through proper placement of the kit and application of sterile gloves. The non dominant hand should be used for cleansing and kept in place to prevent contamination. The sterile dominant hand should be used for insertion.

The nurse prepares to insert an indwelling urinary catheter in a client who is disoriented to time, place, and person and cannot follow directions or commands. Which intervention is most important when inserting the urinary catheter? a) Ensure the client understands the procedure prior to implementation b) Maintain a sterile field and keep the urinary catheter sterile c) Place the catheter supply kit between the client's legs in the center of the bed d) Throw swabs used to clean the perineum directly into the biohazard bin

b) Gloves Personal protective equipment (PPE) is necessary when a client is on contamination precautions (eg, droplet, airborne, contact). A gown is not normally required in an airborne precaution room; however, if contamination is probable (eg, dressing change, contact with bodily fluids), a gown is necessary. The proper removal of PPE limits self-contamination. The exact procedure for donning and removing PPE varies with the level of precautions and location of nursing practice. Gloves should be removed first and promptly after use to prevent contamination of other items or non contaminated materials (options 2). To remove gloves: Grasp the first glove by its palmar surface and pull off inside out. Next, slide fingers of the ungloved hand under the second glove at the wrist and peel off over the first glove. Discard gloves in an infectious waste container. To remove gloves: Grasp the first glove by its palmar surface and pull off inside out. Next, slide fingers of the ungloved hand under the second glove at the wrist and peel off over the first glove. Discard gloves in an infectious waste container. (options 1, 3, and 4) Face shield/goggles, gown, and mask/respirator can be removed after gloves, which are considered the most contaminated piece of PPE. Educational objective: The proper removal of personal protective equipment limits self-contamination. Gloves should be removed first and promptly after use to prevent contamination of other items or non contaminated materials.

The nurse removes personal protective equipment (PPE) after completing a wound dressing change for a client in airborne transmission-based precautions. Which PPE should the nurse remove first? a) Face shield/goggles b) Gloves c) Gown d) Mask/respirator

a) "After applying the ointment, I'll tightly close my eyes and rub the lid for 2-3 minutes." Ophthalmic lubricants (drops, ointment, gel) replace tears and add moisture to the eyes. They are prescribed to treat dry eyes, a common symptom in clients with Sjögren's syndrome, an autoimmune disorder. Administering an ophthalmic ointment by tightly closing the eyes and rubbing the lid for 2-3 minutes can squeeze the ointment out of the eye and cause injury. The client is taught to gently close the eyes for 2-3 minutes to distribute the medication after applying the ointment. (option 2) This statement indicates the client's understanding that when self-administering the medication, the client should squeeze a thin strip of ointment onto the lower eyelid, from the inner to the outer edge, and without letting the tube touch the eye to prevent contamination. (option 3) This statement indicates the client's understanding that when self-administering the medication, the client tilts the head back, pulls the lower lid down, and looks toward the ceiling to help decrease blink reflex. (option 4) Some clients use the ophthalmic ointment at bedtime and the eye drops during the day due to blurred vision that ointments and gels can cause. Educational objective: Teach client the following steps for self-administration of ophthalmic ointments: 1. Perform hand hygiene 2. Tilt the head back, pull the lower lid down, and look upward 3. Squeeze a thin strip of ointment onto the lower eyelid, from the inner to the outer edge 4. Close the eyes gently for 2-3 minutes after applying the ointment

The nurse teaches a client with newly diagnosed Sjögren's syndrome how to self-administer ophthalmic lubricating ointment medication. Which statement that the client makes indicates the need for further teaching? a) "After applying the ointment, I'll tightly close my eyes and rub the lid for 2-3 minutes." b) "I'll squeeze a thin strip of ointment on my lower eyelid, from the inner to the outer edge." c) "I'll tilt my head back, pull my lower lid down, and look upward when administering the ointment." d) "I'll use my ointment at bedtime and my eye drops during the day."

d) Using a syringe, administers the medication in small amounts into the back of the cheek Using a syringe to measure the medication is the most accurate technique to ensure that the proper amount of medication is being administered. The correct procedure for administering oral medication to an infant is to place small amounts of the medication at the back of the cheek, allowing time for the infant to swallow each amount. This technique decreases the risk for choking and ensures that all the medication is consumed. (option 1) Although cup feeding may be a method used to feed infants in specific cases, medication administration requires a more accurate measurement. A syringe can provide an accurate measurement and decrease the risk of waste due to the infant's spitting or drooling. (option 2) Infants have a decreased gag reflex. Dispensing medication onto the back of the tongue would increase the risk for aspiration of the medication. (option 3) It is very important for the infant to receive the entire dose of the medication. Medication should never be mixed in a bottle of formula as the infant may not consume the entire amount. Educational objective: The extrusion reflex and decreased gag reflex in infants less than 4 months old increase the risk for choking and aspiration. Instilling the medication using a syringe at the back of the cheek decreases the risk for choking and ensures that the correct amount of medication is consumed.

The nurse teaches a parent how to administer an oral liquid medication to a 2-month-old client. The nurse knows that the parent understands the teaching then the parent does which of the following? a) Allow the client to sip the medication from the cup b) Expels the medication from a dropper onto the back of the tongue c) Mixes the medication in the infant's bottle of formula d) Using a syringe, administers the medication in small amounts into the back of the cheek

b) Guide suppository along the rectal wall c) Hold buttocks together firmly after insertion d) Position client supine with knees and feet raised e) Use gloved fifth finger for insertion Pediatric administration of rectal suppositories is similar to the adult technique, with a few key modifications due to the small size of a child's colon and varying developmental needs. Age-appropriate explanations and/or distractions should be implemented to reduce distress. Toddlers and infants may benefit from distraction with a toy; preschoolers and older children can be instructed to take deep breaths or count during the procedure. Basic steps for suppository administration include the following: 1. Apply clean gloves and position the client appropriately based on age and size (eg, infant supine with knees and feet raised, older child side-lying with knees bent) (option 4). 2. Lubricate the tip of the suppository with water-soluble jelly. Petroleum-based products can reduce absorption. 3. Insert the suppository past the internal sphincter using the fifth finger if the child is under 3 years (option 5). Use of the index finger may cause injury to the colon or sphincters in children younger than age 3 years. 4. Angle suppository and guide it along the rectal wall. The suppository should remain in contact with the rectal mucosa (and not be buried inside stool) to ensure systemic absorption (option 2). 5. Hold the buttocks together for several minutes, or until the urge to defecate has passed, to prevent immediate expulsion (option 3). 6. If a bowel movement occurs within 10-30 minutes, observe for the presence of the suppository. (option 1) The suppository must be inserted past both the external and internal sphincters for proper placement. If not inserted far enough, it may be expelled before achieving a therapeutic effect. Educational objective: In children younger than age 3 years, suppositories are inserted with the fifth finger of the nurse's gloved hand. Age-appropriate explanations and/or distractions are implemented to reduce stress.

The pediatric nurse is preparing to administer an acetaminophen suppository to an 11-month-old with pyrexia. Which actions are appropriate? Select all that apply. a) Advance past the external sphincter only b) Guide suppository along the rectal wall c) Hold buttocks together firmly after insertion d) Position client supine with knees and feet raised e) Use gloved fifth finger for insertion

d) Contact the health care provider A STAT order indicates that the medication should be given immediately and only one time. A new prescription for the medication must be acquired before the dose can be repeated. The most appropriate action is to contact the health care provider to request an as-needed prescription for pain medication. (option 1) A STAT medication dose was administered and cannot be repeated without a new prescription. (option 2) In most states, the registered nurse (RN) cannot delegate the administration of IV opioids to the licensed practical nurse, and it cannot be administered without a new prescription. (option 3) The Rn can delegate repeat vital sign checks to the unlicensed assistive personnel, but it is not the most appropriate action. Educational objective: A STAT order indicates that a medication is to be given immediately and only once.

The registered nurse (RN) is providing nursing care with a licensed practical nurse and unlicensed assistive personnel. The RN administers hydromorphone 1.5 mg IVP per STAT order to a client with severe abdominal pain. Three hours later, the client rates pain as a 9 on a scale of 0-10 and requests pain medication. What is the most appropriate action for the RN to take? a) Administer the hydromorphone b) Ask the licensed practical nurse to administer the medication c) Ask the unlicensed assistive personnel to take repeat vital signs d) Contact the health care provider

a) Instruct the parent to launder the child's clothing and store it in tightly sealed bags Although full-blown bed bug infestations are uncommon in a school setting, a bed bug brought in on the clothing or possessions of one student could easily "hitch" a ride to another student's home and cause an outbreak there. The most important measure to prevent bed bugs from infesting other students' homes is to prevent the bugs from entering the school in the first place. Laundering clothing in hot water and using the highest temperature setting on a dryer will kill any bed bugs attached to clothes. The clothing should then be stored in tightly sealed plastic bags to prevent additional infestation (option 1). (option 2) A professional pest control company should be brought in to evaluate the classroom/school for bed bugs; treatment with an insecticide may or may not be necessary. (option 3) Sending letters home to parents is premature at this point. After professional pest control personnel evaluate the classroom/school, letters can be sent to inform parents of the findings and any precautions that should be taken. (option 4) Sending the child home is unnecessary and may be perceived as punitive and stigmatizing. Bed bugs do not inhabit humans; this child is not "infested" (seen in children with head lice). Educational objective: The most important measures to prevent bed bugs from getting onto apparel is to launder clothes in hot water, dry them using the highest temperature setting on a dryer, and then store them in tightly sealed plastic bags. This will help to prevent additional bed bug infestation and transportation to other locations.

The school nurse is speaking with the parent of a fourth grade student about a bed bug that was found on the child's sweater. The parent confirms that their home is infested but that the issue is being resolved. Which is the best action by the nurse? a) Instruct the parent to launder the child's clothing and store it in tightly sealed bags b) Instruct the teacher of the child's classroom to use an insecticide spray c) Send letters home to all of the children's parents informing them about the finding d) Send the child home and prohibit school attendance until the infestation has been resolved

a) Ask the spouse to further deceive the client's symptoms The first step in the nursing process is assessment. In this situation, additional information is needed before the nurse can determine the next course of action (option 1). (option 2) The United States' Health Insurance Portability and Accountability Act (HIPPA) and Canada's Personal Information Protection and electronic Documents Act (PIPEDA) prevent release of private, privileged health care information to people who do not need to know it for a client's care. In this case, the nurse is not releasing any information and is obtaining further information to assess the client's condition. In addition, there is not privacy violation in obtaining information that the spouse would know. (option 3) Additional information is required before knowing whether the client needs to be seen in the clinic. The client may need instruction to instead call 911 and go to the emergency department. (option 4) The nurse can ask the client to call, but the client may be unable (eg, seizure, unconscious) or unwilling to do so. In addition, the client may not be aware of signs (eg, acute-onset confusion) that are concerning to the spouse. The situation is unclear (eg, the client may have trouble speaking [ie, stroke symptom]) but may be clarified after the nurse receives additional information from the spouse. Educational objective: The nurse should further assess the situation and gather more information when a spouse calls reporting troublesome symptoms in a client. It is not a violation to obtain information about a client from a knowledgeable source.

The spouse of a client calls the nurse at the clinic and reports that the client is not feeling well and is concerned that something is seriously wrong. How should the nurse respond initially? a) Ask the spouse to further deceive the client's symptoms b) Indicate that privacy rules prevent discussion of concerns with the spouse c) Offer a same-day appointment to the client d) Tell the spouse to have the client call the nurse

d) Child with suspected pertussis who has paroxysms if coughing Paroxysms of rapid coughing that lead to vomiting are a key feature of pertussis infection. Pertussis is a highly contagious disease and requires droplet precautions. It can be deadly if contracted in infancy before vaccination is started. This client should be placed in isolation immediately to prevent the spread of disease. (option 1) Chickenpox is no longer contagious after the lesions have crusted and dried, but this process can take as long as 3 weeks. This client would not require isolation. (option 2) Impetigo is no longer contagious after 24 hours of antibiotics. This client would not require isolation. (option 3) Poison ivy rash is not considered contagious. A person develops the rash only on contact with the urushiol oil itself. The pustules do not contain this oil, and therefore the rash cannot be spread via person-to-person contact. Educational objective: Chickenpox is no longer contagious after the lesions have crusted and dried. Pertussis is a highly contagious disease that requires droplet precautions.

The triage nurse has one isolation room left in the emergency department. Which priority client should be assigned to this room? a) Child with chickenpox for the past 14 days; all lesions are crusted and dried b) Child with impetigo who has been on antibiotics for 3 days c) Child with leg rash secondary to poison ivy exposure d) Child with suspected pertussis who has paroxysms if coughing

b) Discard urine and container, and restart the 24-hour urine collection tomorrow morning Timed urine collection tests are usually done to assess kidney function and measure substances excreted in the urine (eg, creatinine, protein, uric acid, hormones). These tests require the collection of all ursine produced in a specified time period (a crucial step) to ensure accurate test results. The proper container (with or without preservative) for any specific test is obtained from the laboratory. The collection container must be kept cool (eg, on ice, refrigerated) to prevent bacterial decomposition of the urine. Not all of the client's urine was saved during the collection period. Therefore, the nurse or UAP must discard the urine and container and restart the specimen collection procedure. Although a 24-hour urine collection can begin at any time of the day after the client empties the bladder, it is common practice to start the collection in the morning after the client's first morning voiding and to end it at the same hour the next morning after the morning voiding (option 2). (option 1) Adding 250 mL to the total output when the test is completed is not an appropriate action as the actual urine output from the 24-hour period is needed for accurate results. (option 3) To start the collection period, the nurse asks the client to void and discards this specimen (it is not added to the collection container). The 24-hour period starts at the time of the client's first voiding. (option 4) Relabeling the same container and changing the start time from 6:00 AM to 10:00 AM is not an appropriate action. The container would include part of the urine produced in a 28-hour period, and the test results would be inaccurate. Educational objective: It is common practice to start a 24-hour urine collection test at the time of the client's first voiding in the morning. If any urine is discarded by accident during the test period, the procedure must be restarted. All produced urine should be placed in the same container and kept cool (on ice).

The unlicensed assistive personnel (UAP) reports discarding a urine specimen of 250 mL at 10:00 AM by mistake but adding all specimens to the collection container before and after that time. What action should the nurse take? a) Add 250 mL to the total output after the 24-hour urine collection is complete tomorrow morning b) Discard urine and container, and restart the 24-hour urine collection tomorrow morning c) Discard urine and container, have client void, add urine to new container, and then restart test d) Relabel the same collection container, and change the start time from 6:00 AM to 10:00 AM

a) Position client in high Fowler's for a paracentesis related to end-stage cirrhosis d) Position client Trendelenburg on left side if air embolism is suspected e) Position client with arm raised above head for chest tube placement Abdominal paracentesis is used to remove ascitic fluid from the peritoneal cavity in end-stage liver disease (cirrhosis). The client should be positioned in high Fowler's or sat upright to facilitate the flow of fluid to the bottom of the peritoneal cavity, where the needle will be inserted. The client should void prior to the procedure to decrease the risk of bladder puncture (option 1). In the event of an air embolus, the head of the bed should be lowered (Trendelenburg) and the client positioned on the left side; this will cause the air to rise to the right atrium. The health care provider should be notified immediately and the nurse should remain with the client (option 4). Chest tube insertion should be performed with the client's arm raised above the head on the affected side. If possible, the head of the bed should be raised 30-60 degrees to reduce risk of injury to the diaphragm (option 5). (option 2) After a liver biopsy, the client should lie on the right side for a minimum of 2 hours (to apply pressure and splint the puncture site) and then supine for an additional 12-14 hours. the risk for bleeding is increased due to the high vascularity of the liver, but correct positioning reduces this risk. (option 3) During a lumbar puncture, the client is positioned side-lying, with the head, back, and knees flexed. A small pillow may be placed between the legs and under the head for comfort and to maintain the spine in a horizontal position. Following the procedure, the client will be positioned according to the health care provider's prescription (usually supine or with head of the bed elevated 30 degrees). Educational objective: For medical procedures, the nurse should ensure that the client: - Has an empty bladder and is in high Fowler's or a sitting position for paracentesis - Is Trendelenburg on the left side for suspected air embolism - Has the arm raised above the head on the affected side for chest tube insertion - Lies on the right side (for 2 hours) and then supine (12-14 hours) after liver biopsy - Is side-lying with the head, back, and knees flexed for lumbar puncture

Which of these are correct nursing actions related to client positioning? Select all that apply. a) Position client in high Fowler's for a paracentesis related to end-stage cirrhosis b) Position client on left side after liver biopsy c) Position client on side with head, back, and knees flexed after lumbar puncture d) Position client Trendelenburg on left side if air embolism is suspected e) Position client with arm raised above head for chest tube placement

a) Belt restraint used for a confused client who keeps trying to get out of bed but is on bed rest e) Soft ankle restraint to prevent bleeding at the femoral site following cardiac catheterization A physical restraint is a device or method used to immobilize or limit physical mobility or body movement to prevent falls, injury to self or others, or removal of medical devices. The client situation, rather than the device, determines whether it is classified as a restraint. Prescribed orthopedic immobilizes and protective devices used temporarily during routine procedures or examination are not considered physical restraints and do not require authorization for use from a health care provider. Restraints should be used only after less invasive methods have failed and must be discontinued at the earliest time possible once it is safe to do so. The belt restraint is applied at the waist and tied to the bed frame under the mattress with straps using a quick-release knot. It is used to protect a confused or disoriented client who is on bed rest. Although the client can turn, it is considered a restraint because it restricts physical mobility and confines the client to the bed involuntarily (option 1). Soft limb restraints (eg, wrist, ankle) immobilize one or more extremities and are used for the prevention of falls or attempted removal of devices. Following a procedure requiring sedation, clients may require restraints to protect them from disrupting a surgical site or medical device until they are alert enough to follow instructions independently (option 5). Limb restraints should be applied loosely enough that 2 fingers can be inserted underneath the secured restraint. Limb restraints should applied loosely enough that 2 fingers can be inserted underneath the secured restraint. The nurse should closely monitor the peripheral neurovascular status and skin integrity of a client's restrained extremity. (option 2) Elbow restraints used as a protective device temporarily immobilize a child (<30 minutes) to perform a medical, diagnostic (eg, drawing blood), or surgical procedure are not considered a physical restraint. (option 3) The use of full padded side rails in the raised position for clients during a seizure protected them from immediate injury; Thor's rays are not considered a restraint. (option 4) An orthopedic leg immobilizer used to restrict movement and maintain a client's extremity in proper alignment is prescribed for therapeutic purposes and is not considered a restraint. Educational objective: Common physical restraint devices include limb (eg, ankle, wrist) and belt restraints. The client situation, rather than the device, determines whether it is classified as a restraint.

Which situations would require the nurse to obtain a prescription for physical restraints? Select all that apply. a) Belt restraint used for a confused client who keeps trying to get out of bed but is on bed rest b) Elbow restraints used temporarily for a toddler while drawing blood c) Full padded side rails in the raised position for a client during a seizure d) Long leg immobilized used for a client with a fractured tibia e) Soft ankle restraint to prevent bleeding at the femoral site following cardiac catheterization

b) Insert the IV line into the most distal site of the right arm c) Place an appropriate precaution sign above the bed d) Review the medical record for history of mastectomy A modified radical mastectomy includes removal of axillary lymph nodes that are involved in lymphatic drainage of the arm. Any trauma (eg, IV extravasation) to the arm on the operative side can result in lymphedema, characterized by painful and lengthy swelling, as normal lymphatic circulation is impaired by scarring. Therefore, starting an IV line in this arm is contraindicated. The nurse should insert the IV line into the most distal site of the unaffected side (option 2). For client safety, it is also important to ensure documentation of the mastectomy history, place a restricted extremity armband on the affected arm, and place a restricted extremity armband on the affected arm, and place a sign above the client's bed notifying hospital staff of necessary mastectomy precautions (eg, no blood pressure measurements, venipuncture, or IV lines) (options 3 and 4). In general, venipuncture is contraindicated in upper extremities affected by: - weakness - paralysis - Infection - Arteriovenous fistula or graft (used for hemodialysis) - Impaied lymphatic drainage (prior mastectomy) (option 1) The stylet should be advance until blood return is seen (approximately 1/4 inch). If advanced fully, the stylet may penetrate the posterior wall of the vein and cause a hematoma. (option 5) Keeping the affected arm in a dependent position for a long time can increase lymphedema. The client should be reminded that raising the limb helps drainage. Educational objective: IV line insertion is contraindicated on the operative side of clients with a prior mastectomy. Additional contraindications for IV line insertion include weakness, paralysis, or infection of the arm; or presence of an arteriovenous fistula.

While preparing to insert a peripheral IV line, the nurse notices scarring near the client's left axilla. The client confirms a history of left breast cancer and modified radical mastectomy. Which actions should the nurse take? Select all that apply. a) Advance the entire stylet into the vein upon venipuncture b) Insert the IV line into the most distal site of the right arm c) Place an appropriate precaution sign above the bed d) Review the medical record for history of mastectomy e) Teach the client to keep the left arm in a dependent position


Related study sets

Chapter 14.3 Meiosis and Sexual Reproduction

View Set

Ch. 2 Developing the Problem and Using the Literature Quiz

View Set